Sie sind auf Seite 1von 61

SAMPLE QUESTIONNAIRE FOR “ISMS” PHILOSOPHY OF RATIONALIZATION: C - Realism.

Realism concerns with what is


EDUCATION (SOCIAL DIMENSIONS) real, actual. For ideas to be realized, they must be transferred or
demonstrated.
1. The Philippine Elementary School Curriculum gives greater
emphasis on the development of basic skills like reading, writing, and 4. In his class, Teacher M always presents principles and values so as
arithmetic. What is the philosophical basis for this? to encourage his students to examine them and decide for themselves
A. Pragmatism C. Essentialism whether to accept them or not. What kind of philosophy does he
B. Perennialism D. Existentialism practice?
A. Idealism C. Humanism
RATIONALIZATION: C. Essentialism. Essentialism - is an B. Essentialism D. Existentialism
educational theory rooted in classical realism and idealism which
advocates curricular reform stressing the essentials of the basics like RATIONALIZATION: D - Existentialism. Existentialism is a
the 3R's. philosophy that emphasizes subjectivity, freedom and responsibility.
Pragmatism- is regarded as an American philosophy whose various
forms advocate experimentalism, 5. When a teacher emphasizes that man’s sense should be trusted
instrumentalism, functionalism, and practicalism and their curricular because they are the only
offerings imply a wide range of subject areas. way to acquire knowledge, the teacher can be regarded as .
Pereninialism- is a traditional educational theory that puts premium on A. Naturalist C. Empiricist
eternal truth as contained in the "Great Books". B. Realist D. Pragmatist
Existentialism - personalizes knowledge to the individual. The person
chooses the knowledge that he deems is relevant to his process of RATIONALIZATION: C -Empiricism. Empiricism upholds that the
becoming to realize his essence. only source of knowledge is the senses and sense-based experience.
2. Teacher H asks one of her students, “What do you want to become 6. Teacher K views her pupils as unique, free choosing, and
when you grow up?” This responsible individuals. She plans activities where the pupil can
question is an indication of what kind of philosophy? A. Progressivism develop his unique personality. What theory underlies this nature of
C. Existentialism the pupil?
B. Naturalism D. Idealism A. Realism C. Existentialism
B. Essentialism D. Progressivism
RATIONALIZATION: D - Idealism. Idealism is the answer because it
stresses the existence of ideas independent from the material world. RATIONALIZATION: C. Existentialism. Existentialism is the
Ideas that which exist in the mind are the only reality. philosophy of subjectivity of self hood whose fundamental doctrine
proclaims man’s freedom in the accomplishment of his destiny. In this
3. Teacher X has not only explained the concept of Philosophy of case, emphasis is given on the students rather than on the curriculum
Education but also imparted this to her students. This demonstrates content.
what kind of philosophy?
A. Naturalism C. Realism 7. "Learning is the process of retrieving prior learning", this is a
B. Idealism D. Perennialism statement from _. A. Constructivist C. Progressivist
B. Reconstructivist D. Empiricist
1
RATIONALIZATION: A. Constructivist. As defined, Constructivism 11. Thea listened to the advice given by her sister to end the
is a philosophy of learning which asserts that reality does not exist relationship that she has with Gilbert. However, her sister learned that
outside of the human conceptions. It is the individual that construct the advice she has given was not followed and Thea decided to
reality by reflecting on his own experience and gives meaning to it. It continue the relationship. This action of Thea is a manifestation of
is founded on the premise that by reflecting on our own experiences, what kind of philosophy?
we construct our own understanding of the world we live in. A. Essentialism C. Perennialism
Therefore, learning is simply the process of adjusting our mental B. Existentialism D. Humanism
modes to accommodate new experiences.
RATIONALIZATION: B. Existentialism. Existentialism is the
8. Teacher U teaches to his pupils that pleasure is not the highest good. philosophy of subjectivity of selfhood
Teacher's teaching is against what philosophy? and proclaims man’s freedom in the accomplishment of his destiny.
A. Realism C. Epicureanism
B. Hedonism D. Empiricism 12. After finishing the degree in Education, Teacher M learns that
learning never stops. In fact, she accumulates more knowledge after
RATIONALIZATION: B. Hedonism. Epicureanism is just a form of leaving the portal of her alma mater. This typifies what kind of
Hedonism. Hedonism is the general, Epicureanism is the specific. philosophy?
Always select the general answer. A. Constructivism C. Progressivism
B. Perennialism D. Humanism
9. After listening to the homily of the Priest about fidelity, Catherine
has a moment of reflection. Her understanding of the value of fidelity RATIONALIZATION: C. Progressivism. Progressivism connotes
has become deeper as she related this to her past experience. This change, growth and development. It is an active form of philosophy
typifies what kind of philosophy? for it continues to evolve.
A. Constructivism C. Humanism
B. Reconstructivism D. Existentialism 13. Matilda is an advocate of the principle “making the most of your
life”. She is indeed an
RATIONALIZATION: A. Constructivism. Constructivism is a advocate of what kind of philosophy?
philosophy of learning which asserts that reality does not exist outside A. Humanism C. Realism
the human conceptions. It is the individual who construct reality by B. Perennialism D. Existentialism
reflecting on his own experience and gives meaning to it.
RATIONALIZATION: A. Humanism. Humanism is a philosophy that
10. After studying the Principle of Identity, Teacher W asks her stresses to “live life to the fullest”.
students to determine which among the given set of problems
conforms to the said identity. This shows that Teacher W upholds what 14. Teacher X is a very dedicated teacher in the nursery. Her foremost
kind of Philosophy? concern is for students to learn how to adapt themselves in the
A. Perennialism C. Essentialism environment. This shows that Teacher X upholds what kind of
B. Progressivism D. Naturalism philosophy?
A. Naturalism C. Progressivism
RATIONALIZATION: B. Progressivism. Progressivism is a B. Pragmatism D. Rationalism
philosophy of learning by doing things.

2
RATIONALIZATION: A. Naturalism. Naturalism aims to unfold the 18. Principal C shares this thought with his teachers: "Subject matter
child’s potential, not to prepare him for a definite vocation or social should help students understand and appreciate themselves as unique
position but to prepare him to adapt to the changing times and needs. individuals who accept complete responsibility for their thoughts,
feelings, and actions." From which philosophy is this thought
15. Teacher J serves as an inspiration to his students because of his based? / Teacher V teaches her students to appreciate themselves as
efficiency and effectiveness as a teacher. The mind set of his students unique individuals and are
towards him/her is an instance of what kind of philosophy? to accept the responsibility to their feelings, actions and thoughts. She
A. Realism C. Nationalism upholds the philosophy of
B. Idealism D. Constructivism .
A. Perennialism C. Existentialism
RATIONALIZATION: B. Idealism. Idealism is a philosophy which B. Essentialism D. Progressivism
asserts that what’s in the mind is the only reality. In this situation, the
teacher inspires his/her students through the values that he/she RATIONALIZATION: C. Existentialism. Existentialism is the
possessed. Values are absolute are told to belong in realm of ideas. philosophy of subjectivity of selfhood
Hence, these are considered as ideas that exist only in the mind. and proclaims man’s freedom in the accomplishment of his destiny.

16. John learns well through active interplay with others. His learning 19. Teacher H class is engage in problem solving activity which in a
increases when engaged in activities that has meaning in him. Which way is a reflection of the personal and social experiences. The purpose
philosophy can describe this? of this activity is for the students to acquire the skills that can help
A. Progressivism C. Realism them in solving their own real-life problems. What philosophy can
B. Idealism D. Empiricism best describe this?
A. Realism C. Progressivism
RATIONALIZATION: A. Progressivism. Progressivism is a B. Idealism D. Existentialism
philosophy of hands of learning or experiential learning. “Learning by
doing”. Learning is based from the questions of one’s experience of RATIONALIZATION: C. Progressivism. Progressivism connotes
the world. Hence, it is the learner himself who thinks, solves, and growth and development. In this manner learning comes best when
gives meaning through his individual experiences. things are being experienced, that is learning by doing
things. It involves participation of the students and allows them to
17. Your teacher is of the opinion that the world and everything in it exercise freedom.
are ever changing and so teaches you the skill to cope with the
changes. Which in his governing philosophy? 20. In the Social Science class of Teacher G, students identify the
A. Experimentalism C. Idealism various social and economic problems that require urgent solutions.
B. Existentialism D. Realism They not only discuss the ways to address it but also agreed to
participate in solving them. What kind of philosophy does this class
RATIONALIZATION: A. Experimentalism. Experimentalism believes uphold?
that things are constantly changing. It is based on the view that reality A. Naturalism C. Progressivism
is what works right now and that goodness comes from group B. Constructivism D. Reconstructivism
decisions. As a result, schools exist to discover and expand the society
we live in. Students study social experiences and solve problems. RATIONALIZATION: D. Reconstructivism. Reconstructivism covers
the underlying factors that constitute reality or society. In this regard,
3
students are encourage to become involved in the problems whether 24. Mr. Cruz exemplifies an ideal teacher and son. He is competent in
political, social, or economical that confront the society and be able to his field based on the various recognitions that he received from his
arrive at solutions in order to reconstruct society. school. As a dutiful son, he assumes all the responsibilities of raising
his family (this includes his parents, siblings, and their own families) –
21. A curriculum should only include those that have survived the test giving all their needs and wants. The fulfilment of these is the neglect
of time and combine the symbols of literature, history, and of the responsibilities to himself. This attitude of Mr. Cruz is against
mathematics. Thus, curriculum like this contains values that are what kind of philosophy?
constant and universal. What philosophy describes this kind of A. Existentialism C. Pragmatism
curriculum? B. Constructivism D. Humanism
A. Idealism C. Humanism
B. Perennialism D. Essentialism RATIONALIZATION: D. Humanism. Humanism stresses personal
culture, individual freedom, and development (the best way towards
RATIONALIZATION: B. Perennialism. Perennialism maintains that full and rich lives).
education involves confronting the problems and questions that have
challenged people over the centuries. Thus there is a need to study 25. Teacher R ensures to put a certain amount of his monthly earnings
classical tradition of great books. in the bank. At the end of the school year, he used all his savings in
visiting places rich of cultural heritages rather than of buying
22. Who among the following puts more emphasis on core expensive clothes, jewellery, and latest gadgets. Favoring the former
requirements, longer school day, longer academic year and more action over the latter exhibits that Teacher R follows what kind of
challenging textbooks? philosophy?
A. Perennialist C. Progressivist A. Existentialism C. Behaviorism
B. Essentialist D. Existentialist B. Essentialism D. Progressivism
RATIONALIZATION: B. Essentialist. Essentialism is an educational RATIONALIZATION: A. Existentialism. Existentialism is a
theory rooted in classical realism and idealism which advocates philosophy that gives a person the freedom to choose. It is a
curricular reform stressing the essentials of the basics like the 3R's. philosophy of subjectivity.
23. “Education is a continuous process of experiencing and of visiting 26. Teacher R ensures to put a certain amount of his monthly earnings
or reorganizing in the bank. At the end of the school year, he used all his savings in
experiences “according to a Progressivist. What does it mean? visiting places rich of cultural heritages rather than of buying
A. Education begins and ends in school. expensive clothes, jewellery, and latest gadgets. This shows that
B. Education takes place anytime and anywhere. C. Education Teacher R is an advocate of what kind of philosophy?
happens formally and informally. A. Pragmatism C. Naturalism
D. Education goes on throughout life. B. Perennialism D. Essentialism
RATIONALIZATION: D. Education goes on throughout life. RATIONALIZATION: B. Perennialism. Perennialism views education
Progressivism connotes growth and development. It is an active form as a recurring process based on eternal truths. Good education
of philosophy for it continues to evolve. involves a search for and understanding of truth which can all be
found in the great works of civilization.

4
27. Teacher X rates the art work of her students not just on the merit of 30. Increase of time allotment in English, Math and Science in the
its appeal to the senses but also considers its uniqueness and the Elementary level under the New Elementary School Curriculum
responsibility that every student has given in accomplishing the task. (NESC) as provided in DECS Order No. 1, s. 1993 contributes in the
This practice shows how Teacher X upholds this kind of philosophy: culturation of basic literacy by providing the needed essential skills.
A. Realism C. Idealism The related philosophy is .
B. Reconstructivism D. Existentialism A. Realism C. Idealism
B. Essentialism D. Perennialism
RATIONALIZATION: D. Existentialism. Conceives philosophy as
something that is felt by an individual because it’s concrete in itself or RATIONALIZATION: B. Essentialism. Essentialism is an educational
based on what is concrete. There is also personal involvement in this theory rooted in classical realism and idealism which advocates
kind of philosophy. curricular reform stressing the essentials of the basics like the 3R's.
28. In line with the philosophy of Reconstructivism, which of the 31. Teacher A believes that the learner is the product of his
following should be given emphasis in teaching? environment. Sometimes he has no choice. He/she is determined by
A. To seek a better position in the society his environment.
B. To compare oneself with the less fortunate A. Rationalist C. Existentialist
C. To become economically self-reliant B. Behaviorist D. Progressivist
D. To designate one’s superiority over the others
RATIONALIZATION: B. Behaviorist. A behaviourist believes that
RATIONALIZATION: C. To become economically self-reliant. human behaviour can be best explained in terms of responses to
Reconstructivism aims to achieve social changes. This is a philosophy external stimuli. Also, education can be best achieved by modifying or
that entails the identification of social problems, finding solutions for changing student behaviours in a socially acceptable manner through
this and be a part of the solution. the arrangement of the conditions for learning. The control is obtained
not by manipulating the individual, but the environment.
29. Joining organizations or clubs in school is requisite to granting of
academic distinction under DECS Order No. 65, s. 1998. This shows 32. The curriculum is viewed as a means of developing desirable
school community connection reflected in activities geared towards habits. It is recommended that the way /means to form these habits is
society's needs. What philosophy is related to this? / Activities planned through the mastery of organized subject matter. Which philosophy is
by behind this educational view?
school clubs or organizations show school-community connection A. Naturalism C. Realism
geared towards society's needs. What philosophy is related to this? B. Idealism D. Pragmatism
A. Realism C. Existentialism
B. Progressivism D. Reconstructivism RATIONALIZATION: C. Realism. This educative process is viewed
mainly as the transmission of information and knowledge. The
RATIONALIZATION: D. Reconstructivism. Reconstructivism aims to curriculum has the function of forming the body and this
achieve social changes. This is a philosophy that entails the interrelationship gives rise to the dictum "sound mind in a sound
identification of social problems, finding solutions for this and be a body."
part of the solution.
33. Teachers in school perform the role and responsibility of parents in
the development and education of the child. This stems from the belief
5
that the home is the primary agency in the education of the individual. c. Do not lend him anymore � � ✔
Thus, teachers are regarded as surrogate parents along the loco d. Ask for a collateral for the cash he is loaning
parentis principle. Which philosophy espouses this view of education? 3. Periodic checks on student seatwork with a smile and pat on the
A. Pragmatism C. Idealism shoulder effectively reinforce good study habit is an example of
B. Naturalism D. Realism __________
a. Discrimination reinforcement
RATIONALIZATION: B. Naturalism - the school is seen as the
b. Variable-ratio schedule
extension of home and teachers are deemed "surrogate parents".
c. Continuous reinforcement � � ✔
34. This philosophy believes that students need a passionate encounter d. Fixed interval and variable-interval schedule
with the positive and negative phases of life like the joy and agony of 4. A person strives to work at a given task because of a need. Which of
love, desirability of life, the inevitability of death, the anguish of the following situations can make a person strive to meet his needs?
freedom and the consequences of choices and actions. Which a. Minimize the unpleasant consequences of student involvement �
philosophy advocates this view in education? � ✔
A. Existentialism C. Essentialism b. Utilize your own opinion as teacher in making final decisions in the
B. Perennialism D. Realism classroom
RATIONALIZATION: A. Existentialism - it is a modern school of c. Use unfamiliar materials as examples in order to initially arouse
thought that grew out of the thoughts of European philosophers , their curiosity
particularly from Soren Kierkegaard, who believes that the central d. Ask pupils to submit test questions or reactions which you can
problem humanity is facing is the ability to cope with its existence. select topics
5. The singing of the National Anthem in schools is an offshoot of the
philosophy of _________
1. As a parent and at the same time a teacher, which of the following
a. Nationalism � � ✔
will you do to show your cooperation to a PTA project in your school
c. Naturalism
to be financed with the proceeds of the sales of the school canteen
b. Pragmatism
where food prices are little bit higher?
d. Socialism
a. Bring food for you and your children, but always make it a point to
6. The environment in order to facilitate, learning must be interactive.
buy in the school canteen. � � ✔ Which of the following best typifies this kind environment?
b. Buy all you food in the school canteen but request for a discount a. The child goes out and discovers for himself some rock or fossil
c. Bring food enough for you and your children but do not eat in the
� � ✔
canteen
b. The child listens to a lecture on fossils given by the teacher
d. Buy all your food from the school canteen even if you cannot afford
c. The child summarize the section on fossils in his science textbook
to do everyday.
d. The child copies a list of facts concerning fossils on the blackboard
2. How can you help a habitual borrower of money get rid of his
7. Social development means the acquisition of the ability to behave in
habit?
accordance with __________
a. Let him do something for you in return for the money you lent him
b. Direct him to others a. Social expectation � � ✔
c. social insight

6
b. Stereotyped behavior regional school
d. universal norms b. To maintain English as a second language
8. When an adolescent combines ability to use deductive and inductive c. To make Filipino the sole medium of instruction � � ✔
reasoning in the construction of realistic rules that he can respect and d. To make Filipino the national Language and medium of instruction
live by, how does he perceive his environment? and communication
a. He views the world from his own perspective 14. The tendency to emphasize so much on school beautification to the
b. He interprets events from a limited view detriment of pupils performance illustrates the
c. He views events apart from himself and other people � � ✔ a. Filipino’s lack of seriousness
d. He views the world and himself through the eyes of other people b. Filipino’s lack of reflection
9. Who introduced the technique of using the drawing of a man as a c. Filipino’s sense of humor
measure of intelligence? d. Filipino’s love for “porma” � � ✔
a. Aristotle 15. Which is NOT a characteristics of democratic discipline?
b. Herbart a. Child has opportunity to express his/her opinion
c. Good Enough � � ✔ b. Child given punishment is related to the misdeed
d. Binet c. Child understands the meaning of rules
10. Which Republic Act provides government assistance to students d. Child obeys blindly � � ✔
and teachers in private education? 16. Who among the following stressed the processes of experience and
a. RA 7784 problem solving?
b. RA 6728 � � ✔ a. Dewey � � ✔
c. RA 7836 b. Aristotle
d. RA 6675 c. Hegel
11. The authoritarian setting in the Filipino home is reinforced by a d. Plato
classroom teacher who: 17. Which of the following reasons of measuring student achievement
a. Encourage pupils to ask questions is NOT valid?
b. Prescribes what pupils should do � � ✔ a. To prepare feedback on the effectiveness of the learning process
c. Is open to suggestions b. To certify that students have attained a level of competence in a
d. Ask open ended questions subject area
12. Who among the following believes that learning requires c. To discourage students from cheating during test and getting high
disciplined attention, regular homework, and respect for legitimate scores � � ✔
authority? d. To motivate students to learn and master the materials they think
a. Essentialist � � ✔ will be covered by the achievement test
b. Perennialist 18. Which characterizes the perfectionist type of students?
c. Progressivist a. Does not volunteer or initiate
d. Reconstructionist b. Give up easily
13. The Constitutional provision on language has the following aims c. Rarely complete tasks
EXCEPT: d. Often anxious fearful or frustrated about quality of work � �
a. To make the regional dialect as auxilliary media of instructions in ✔
7
19. When a school decides to work on a thematic curriculum which d. Encourage the other teachers and students to hold a demonstration
should be out of the picture? to oust the superior
a. Peer collaboration 25. Pick out the situation that illustrates the duty of a new teacher to
c. team teaching the state:
b. Integration a. Take a long vacation which she firmly believes she deserves after
d. competition � � ✔ four years of diligent study before taking the examination for teachers
20. Teacher wants to teach his pupils the technique on reading for b. Apply for teaching job where eligibility is not required to gain
information. Which technique should be used? teaching experience before taking the teachers board examination
a. Text structure c. Prepare for the wedding she and her boyfriend have long planned to
c. Story map able to raise a family with children which they plan to rear as good
b. Prior knowledge citizen of our country
d. SQ4R � � ✔ d. Take the licensure examination for teacher and an oath to do her
21. In instructional planning, which among these three: unit plan, best to help carry out the policies of the state � � ✔
course plan, lesson plan is (are) most specific? _________ plans. 26. Parents are up in arms on the telephone bills that pay for sex calls.
a. Course and lesson What is the solution to this problem?
c. lesson � � ✔ a. The telephone company is to blame for this
b. Course b. The government restriction have no teeth
d. unit c. Parents allow this to make their children modern
22. The use of drills in the classroom is rooted on Thorndike’s law of d. Parents, school and students should discuss this openly � � ✔
a. Readiness 27. Cooperative are encouraged is as many groups as possible. What
c. effect agency controls the different cooperatives?
b. Exercise � � ✔ a. Security and Exchange Commission � � ✔
d. belongingness c. Commission on Audit
23. Positive interdependence as an element of collaborative learning b. Department of Local Government
means that the students must: d. Bureau of Cooperative
a. Learn to depend on each other to achieve a goal � � ✔ 28. Society and media know drinking starts off drug addiction. What
b. Depend on the diligent students should be discussed in schools?
c. Help one another in the individual test for everyone to pass a. Drug addiction has been traced to drinking wine � � ✔
d. Be grouped heterogeneously b. Nobody drinks at home except father
24. Which of the following measures should a teacher do to a principal c. TV ads show drinking is a source of fellowship
whom she would like to file a case of sexual harassment without d. High taxes on liquor will be deterrent to eventual drug use
violating the relationship of the teacher and her superiors? 29. Cooperatives have branched out to consumer cooperatives.
a. Present the case before competent authority and prepare to prove Schools have included the concepts of cooperatives. Where is it
the charge � � ✔ practiced?
b. Write an anonymous letter to a higher school official to denounce a. School book stores
the superior c. Schools uniform purchases
c. Call a parent-teacher meeting and denounce the superior
8
b. School canteen � � ✔ c. Jury trial
d. Class stores b. Composite report � � ✔
30. A student collapsed in her social studies class. It was found out d. Agenda
that he did not eat her lunch. What principle is shown in the situation 36. In the formulation of classroom regulations, which of the
a. Psychological needs following should a teacher refrain from doing?
c. Somatotonic a. State classroom regulation as clearly as possible
b. Physiological need � � ✔ b. Enlist student aid in the formation of classroom regulation
d. Safety need c. Enforce classroom regulations consistently and fairly
31. The main function of a philosophy of education is to: d. Teacher and the class should make as many regulations as possible
a. Aid the learner to build his own personal philosophy � � ✔
b. Reconsider existing educational goals in the light of society’s needs 37. Zero standard deviation means that:
c. Provide the academic background prerequisite to learning a. The students scores are the same � � ✔
d. Define the goals and set the direction for which education is to b. 50% of the scores obtained is zero
strive � � ✔ c. More than 50% of the score obtained is zero
32. Which technique (s) enable (s) a teacher to identify and eventually d. Less than 50% of the scores obtained is zero
assists students with interpersonal difficulties? 38. Which is the least authentic mode of assessment?
a. Anecdotal record a. Paper-and-pencil test in vocabulary � � ✔
c. Cumulative record b. Oral performance to assess student’s spoken communication ski8lls
b. Personal inventory c. Experiments in science to assess skill in the use of scientific
d. Sociogram** methods
d. Artist production for music or art subject
33. Teachers and students can participate in levels of computer use.
Give the order of computer use from simplest to complex? 39. In what period of a child is physical growth fastest?
a. Computer competency, computer literacy, competency expertise a. Prenatal period
b. Computer literacy, computer expertise, computer competency c. Early childhood **
c. Computer competency, computer expertise, computer literacy b. Early adolescence
d. Computer literacy, computer competency, computer expertise � d. Prenatal and early adolescence
� ✔ ✔ 40. How does fear affect the voluntariness of an act?
34. Which one is considered the “Brain” of the microcomputer? a. Makes the act involuntary � � ✔
a. CPU � � ✔ c. Increases voluntariness
b. Software b. No effect at all
c. Video Screen d. Lessens but not destroy voluntariness
d. Keyboard 41. A group of people asserts that their culture is superior to another.
35. A group activity wherein one group representative presents the This exemplifies:
output to the bigger group rather than individual pupils presenting the a. Cultural gap
output is known as: c. norm conflict
a. Consensus decision
9
b. Cultural conflict photocopied term paper provide which Filipino traits?
d. Ethnocentrism � � ✔ a. Anticipation
42. A test consist of a graph showing the relationship between age and c. Pakikisama
population. Follow a series of true-false items based on the graph. b. Ambivalence
Which type of test does this illustrate? d. Lack of discipline � � ✔
a. Laboratory exercise 48. “ No pain, no gain.” This means that
c. Performance a. Only those willing to carry the crosses that life impose can share the
b. Problem solving joy of life � � ✔
d. interpretative � � ✔ b. One should be penitent every Friday by carrying his cross
43. Which curricular move served to strengthen spiritual and ethical c. The more suffering in this life, the more one is assured of heaven
values? d. One should look for suffering to save himself/ herself
a. Integration of creative thinking in all subject 49. Which thrust on value formation is meant to help the students
b. Reducing the number of subject areas into the skill subject make use of their thinking and scientific investigation to decide on
c. Introduction of Value Education as a separate subject area � � topics and questions about values?
✔ a. Value inculcation
d. Re-introducing Science as all subject in Grade 1 c. Value clarification � � ✔
44. Which computer seems to have the most potential for the b. Analysis
classroom? d. Moral development
a. Main frame computer 50. Which interactive teaching should be AVOIDED?
c. Microcomputer � � ✔ a. Using “put down” strategy � � ✔
b. Minicomputer c. Asking more divergent questions
d. LPC b. Using multiple response strategy
45. A teacher notices glaring wrong pronunciation of vowel sounds d. Asking more evaluative questions
among her students necessitating more practice. Which of the 51. Rights which can not be renounced or transferred because they are
following activities would be a most help? necessary for the fulfillment of man’s primordial obligations are
a. Dictionary use called:
c. Assignments a. Alienable rights
b. Review c. Inalienable rights � � ✔
d. Drill � � ✔ b. Perfect rights
46. When storage device is significantly more efficient in holding d. Acquired rights
information 52. In mastery learning the definition of an acceptable standard of
a. Hard disk � � ✔ performance is called:
c. Floppy disk a. Alienable rights
b. Software c. Inalienable rights
d. Audio cassette b. Perfect rights
47. The Filipino tendency to resort to the easy way out from a term d. Acquired rights**
paper as a course requirement by hiring a ghost writer or by passing a
10
53. A negative discrimination index means that: ✔
a. More from the lower group answered the test items correctly � d. Belittling the re-numeration one gets from teaching
� ✔ 59. In writing performance objective which word is NOT acceptable?
a. Manipulate
b. The items could not discriminate between the lower and upper
c. Delineate
group
b. Integrate
c. More from the upper group answered the test item correctly
d. Less from the lower group got the test item correctly d. Comprehend � � ✔
54. Your teacher is of the opinion that the world and everything in it 60. When is giving praise INEFFECTIVE? When IT?
are ever changing and so teaches you the skill to cope with the a. Uses the accomplishment of peers as the context for describing a
changes. Which in his governing philosophy? student’s present accomplishment � � � ✔
a. Experimentation b. Provides information to student’s about their competence and the
c. Realism value of their accomplishment
b. Existentialism � � ✔ c. Focuses students attention on her own task relevant behavior
d. Idealism d. Shows spontaneity, variety and other signs of credibility
55. For brainstorming to be effective which one should be out? 61. Which statement applies when scores distribution is negatively
a. Making use of the others ideas shared skewed?
c.Non-threatening atmosphere a. The mode corresponds to a lower value
b. Teacher’s judge mental attitude � � ✔ b. The median is higher that the mode
c. The mode and median are equal
d. Openness to idea
56. Which statement on spaced and massed learning is CORRECT? d. The mean corresponds to a high value � � ✔
a. Massed learning is better than spaced learning 62. The use of the process approach gives the student the opportunity
b. Spaced learning is better than massed learning � � ✔ to:
a. Learn on their own
c. Massed learning is as effective as spaced learning
c. Make use of laboratory apparatuses
d. Both massed learning and spaced learning are not effective
57. A teacher is a facilitator of learning and of the development of the b. Apply the scientific method � � ✔
youth. Which practice is NOT in keeping with his role as facilitator? d. Learn how to learn
a. Considers the multiple intelligences of learners 63. A comprehension skill of higher level which may be inferred or
b. Humiliates misbehaving pupils � � ✔ implied from reading is
a. Picking out the main idea
c. Dialogs with parents and with other members of the community
c. Following direction
d. Keeps himself abreast with educational trends
b. Noting specific details
58. Which one indicates a teacher’s genuine enthusiasm and pride in
teaching? d. Drawing conclusion � � ✔
a. Sticking to teaching for the moment that there are no better offers 64. In the Preamble of the Code of Ethics of Professional Teachers,
b. Telling everyone that he went to teaching for there was no other which is not mentioned about teachers?
choice them a. Dully licensed professionals
c. Engaging himself in continuing professional education � � c. LET passers � � ✔

11
b. Posses dignity and reputation c. Thinking, feeling and acting being � � ✔
d. With high moral values b. Maternal and an acting being
65. What does a skewed score distribution mean? d. Spiritual and maternal being
a. The scores are concentrated more at one end or the other end ** 71. Both Muslim and Christian value marriage but the Muslim
b. The mode, the mean and the median are equal practices polygamous marriage while the Christian practices
c. The mean and median are equal monogamous marriage. What is this called?
d. The scores are normally distributed a. Cultural relativism � � ✔
66. Which is implied by a negatively skewed score distribution/ c. Ethical relativism
a. The scores are evenly distributed from the left to the right b. Acculturation
b. Most pupils are underachieves d. Enculturation
72. Teacher wants to compare 2 concepts. With which technique can
c. Most of the scores are high � � ✔
accomplish this best?
d. Most of the scores are low
a. K-W-L technique
67. A teacher discovers that a product of a certain bottling company
c. Spider web
brings about damage to teeth. Much as he wants to share the products
of his research, he could not because of harassment from all sides. b. Venn diagram � � ✔
Which teacher’s right is violated? d. Histogram
a. Right to property 73. To build a sense of pride among Filipino youth which should be
done?
c. Academic freedom � � ✔
a. Re-study our history and stress on our achievements as a people
b. Right to one’s honor
d. Right to make a livelihood � � ✔
68. Why can the calculator do arithmetic? Because b. Set aside the study of local history
a. A computer inside the calculator tells it how � � ✔ c. Re-study our history from the perspective of our colonizers
d. Replace the study of folklores and myths with technical subjects
c. A typewriter inside does inside does it
74. When necessary conditions are present, the use of inductive
b. A watch inside direct it
method is preferred because
d. A TV inside shows it
69. On which constitutional provision is the full or partial integration a. There is greater active participation on the part of pupils � �
of capable deaf and blind students in the classroom based? The ✔
provision on b. It gives the teacher more time to rest
a. Providing citizenship and vocational training to adult citizen c. It needs only few instruction materials
b. Protecting and promoting the rights of all citizen to quality d. Academic time is used wisely
75. Which is the best reason why teacher begins a lesson in Math by
education � � ✔
checking and reviewing on the previous day’s assignment and
c. Academic freedom provides practice and drills?
d. Creating scholarship for poor and deserving students a. Check if parents guide their children in the making of assignment
70. Teaching in the cognitive, psychomotor and affective domains is b. Make sure that the students understand the pre-requisite skills of the
based on the concept that the learner is a:
lesson � � ✔
a. Moral and feeling being

12
c. Prepare the students for the mastery test c. Prepare students for law making
d. Make learning interesting and enjoyable for students d. Make constitutional experts of the students
76. Which is a selective reading technique meant at getting at 82. Some students who are high in the scholastic aptitude test have
important facts very fast? failed in college. Some who are below the standards set for admission
a. Skim reading but who for various reasons were admitted, attained satisfactory
c. Oral reading standings. This proves that
b. Scanning � � ✔ a. Human beings are certainly predictable
d. Silent reading b. Admission tests are not accurate, hence should not be used
77. For counseling to be successful which assumption must be c. Aptitude tests do not measure all factors important for success �
AVOIDED? � ✔
a. The environment must provide assurance of confidentiality d. Aptitude test can be perfectly relied on
b. The student is willing to participate in the process 83. If the teachers pattern in questioning consists of calling on a
c. The counselor must be able to relate to the student student then asking the question
d. The counselor tells the student what to do � � ✔ a. All students may be encouraged to participate
78. Which technique is most appropriate when a teacher wants a group b. The student called to answer may be able to think well of his
to agree on a plan of action? answer
a. Composite report c. The rest of the class may just dictate the answer
c. Agenda d. The rest of the class may not engage themselves in thinking of the
b. Symposium answer � � ✔
d. Consensus decision making � � ✔ 84. In order to avoid disgrace, a pregnant, unmarried woman takes
79. Which term applies to the search for related literature by drug to induce abortion. Is she morally justified to do that?
computing access of data bases of discs kept in libraries? a. Yes, it can save her and child from disgrace when he grows up
a. On line research b. No, the unborn child can not be made to suffer the consequences of
c. Compact discs computer research � � ✔ the sins of his parents � � ✔
b. Manual research c. No, the act of inducing abortion is bad in itself
d. Computer research d. No, it is better to prevent the child from coming into the world who
80. Which best indicates the effectiveness classroom activities? will suffer very much due to the absence of a father
a. The laughter and enjoyment of students 85. In which way does heredity affect the development of the learner?
b. The application of concept learned in daily life � � ✔ a. By placing limits beyond which the learner can not develop � �
c. The utilization of varied techniques and approaches ✔
d. The variety of instructional materials used b. By compensating for what environment fails to develop
81. The main purpose of the compulsory study of the Constitution in c. By blocking the influence of environment
Philippine schools is to d. By providing equal potential to all
a. Develop the students into responsible thinking citizens � � 86. The cultivation of reflective and meditative skills in teaching is an
✔ influence of
b. Acquaint students with the historical development of the Philippine a. Taoism
Constitution c. Confucianism
13
b. Shintoism 92. Section 5, article XIV, of the Constitution states that academic
d. Zen Buddhism � � ✔ freedom shall be enjoyed in
87. A child refuse to obey orders or displays negativism as a a. Public assemblies
development trait. How may you best handle him? c. All levels of learning
a. Take every opportunity to praise him for every positive attitude b. State colleges and universities
displayed � � ✔ d. All institution of higher learning � � ✔
b. Detain him after office hours for him do to what he has been 93. A teacher who subscribes to the pragmatic philosophy of education
ordered to do believes that experience should follow learning in her teaching, she
c. Insist on compliance to the same degree required of pupils therefore exerts effort in
d. Avoid giving him orders if you do and he objects take back the a. Encouraging learners to memorize factual knowledge
order b. Equipping learners with the basic abilities and skills
88. Which term refers to the collection of students products and c. Requiring learners full mastery of the lesson
accomplishment for a period of evaluation purposes? d. Providing learners opportunities to apply theories and principles �
a. Portfolio � � ✔ � ✔
c. Anecdotal record 94. Freud expounded that there is a period when young boys
b. Observation report experience rivalry with their father for their mother’s affection. This
d. Diary is
89. For comparing and contrasting which graphic organizers is most a. Oedipus complex � � ✔
appropriate? c. Achilles syndrome
a. Cycle b. Electra complex
c. Story map d. Cassandra syndrome
b. Web 95. Education is a life long process. This simply means that education
d. Venn Diagram � � ✔ a. May take place formally or informally to enable the individual to
90. If a resilient child with superior intelligence is reared in a poor grow
environment the probable outcome would be b. May take place anywhere and any time the individual so desires
a. No change in IQ because environment deprivation has nothing has c. Is a continuous process of experiencing and reorganizing
nothing to do with intelligence experiences � � ✔
b. Slight change in IQ although he can overcome frustration and d. Take place in the school where the individual is exposed, self
obstacle � � ✔ contained experiences
c. Mental retardation since he is culturally deprived 96. The tendency to imitate elders is very strong in the early childhood
d. Great change in IQ because he is culturally deprived stage. Teachers should therefore be very good
91. Which of the following is usually considered the most important a. Counselors
factor in a child’s observable classroom behavior ? c. Disciplinarians
a. Intelligence b. Role models � � ✔
c. Self concept � � ✔ d. Facilitators of learning
b. Heredity 97. How is Values Education offered in the National Secondary
d. Cultural background Education Curriculum?
14
a. Emphasized in Science and Technology b. Intelligence hinges in physical structure
b. As a separate subject � � ✔ c. Heredity has a part in determining intelligence � � ✔
c. Integrated in all subject areas d. Intelligence is determined partly by pre-natal nutrition
d. Integrated with Technology and Home Economics 103. Which of theses philosophers is reflective of that of Dewey’s
98. The NSEC orients secondary education to which stresses the development of an individual capable of reflective
a. The teaching of the national symbols thinking specifically that of being able to solve the problem he faces
b. The development of competencies and values for social living � individually or collectively
� ✔ a. Disciplinarianism c. Experimentation � � ✔
c. Health values development b. Developmentalism d. Rationalism
d. National development requirement and reflects research based 104. Which of the following abilities is stressed by humanistic
direction education?
99. The child cannot distinguish abstracts during the sensory motor of a. Learn the different philosophies of education
development. Which of these techniques should a teacher apply to b. Develop man into a thinking individual
accommodate learning? c. Enjoy the great works of man such as the classics
a. Make use of individualized instruction d. Make man distinctly civilized educated and refined � � ✔
b. Explain the lesson very well 105. An appreciation lesson is one that is designed to lead the class to
c. Utilize concrete objects to clarify concept conduct and enjoy something. Which of the following statements
d. Provide variety of educational toys � � ✔ closely approximate the meaning of the above?
a. An appreciation lesson should be a lesson in values
100. Which of these systems of learning includes ways and methods
b. Appreciation lessons help pupils weigh and clarify values
which are used in preserving and building certain within cultural
c. One cannot fully appreciate what one does not understand or enjoy
communities?
d. A teacher should plan lessons that will guide children to appreciate
a. Non-formal learning
c. Cultural learning ** what is beautiful � � ✔
b. Multi-level learning 106. Which of the following is the best time for a teacher to set up
d. Indigenous learning routine activities that will contribute to effective classroom
management?
101. Which of the following statements is TRUE in the use of a. As soon as the students have established
experiments and demonstrations in teaching science c. During his homeroom days
a. It is valuable if used in the context of a lesson that related b. Daily at the start of the session
observation to other information � � ✔ d. On the very first day of school � � ✔
b. It should be encouraged in elementary school since the concept the 107. In large classes where little of the work pupils can be
encompass are difficult for your children individualized, the most effective and practical ways to individualize
c. It is as valuable as teaching by lecturing instruction is to
d. It is less valuable than teaching through inquiry and discussion a. Devise group activities which afford every pupil an opportunity to
102. Identical twins are more alike than fraternal twins. Which of the
work at his own � � ✔
following statements/ principle is supported by this?
b. Give the pupils freedom to launch individual projects
a. Environment affects both fraternal and identical twins
15
c. Assign homework and check it regularly 113. Which of the following examples illustrate the use of questions to
d. Assign program material for out-of-class hours focus pupil attention on the key points of the lesson?
108. Which of these is the MOST important principle that a teacher a. Why are machine made goods cheaper than those made by hand?
should follow in initiating program for positive reinforcement? � � ✔
a. Make sure the reward comes immediately after the appropriate b. What is Rizal Park known for?
behavior � � ✔ c. Have you ever enjoyed watching the clouds on a bright day?
b. Punish negative behavior and reward positive behavior d. Who came while I was writing on the blackboard?
c. Provide regular opportunity for socially acceptable behavior 114. The new teacher entered a noisy classroom. She shouted
d. Consider peer approval and recognition immediately at the students desperately trying to get order and
109. The trend of focusing attention on the child’s interests, abilities discipline. Since then the teacher has not controlled the class. Which is
and needs and on the improvement of community living necessitate the most probable cause of the teacher’s failure?
the use of the a. The students reaction to the teacher is the consequence of her
a. Discovery approach behavior
c. Integrative approach b. Rules are not defined and procedures to sustain order is not put into
b. Conceptual technique place
d. Project method � � ✔ c. The new teacher wants to show the class who is authority
110. The best way the teacher can see the appropriateness of an d. The class wants to test the ability and patience of the teacher. �
instructional materials is to � ✔
a. Consider its technical quality 115. The educational implementation of research findings relative to
c. Consider its availability the ability of dull learners and bright learners to organize and
b. Try it out before using it in class � � ✔ generalize is for teacher
d. Consider its cost a. To make the bright learners guide the dull ones in learning
111. Tasks analysis involves the breaking down of a learning task into generalize
subtasks or sub skills. Given a task to retell a story, which of the b. To make the bright learners to generalize and the dull ones to
following skills is NOT needed? memorize
a. To disseminate information � � ✔ c. To give the dull learners more concrete experiences to serve as basis
c. To identify topic sentences for generalizing
b. To outline a selection d. To give both the dull and bright learners concrete and abstract
d. To arrange events in sequence experiences to serve as basis for generalizing � � ✔
112. You are assigned to teach students with varied abilities. You want 116. Which of the following will you do the FIRST to establish good
to teach a more homogenous grouping. Which type of grouping will class management?
tend to benefit your students? a. Discuss the required rules for proper class behavior � � ✔
a. Mixed ability grouping b. Discuss the work plan for the year
c. With-in class ability grouping � � ✔ c. Prepare a seat plan
b. Low ability group d. Train the class in the distribution of material
d. High ability grouping 117. A student was diagnosed to have a high IQ but is falling in his
academic subject. What should the teacher do to help him?
16
a. Talk to his parents 123. Who expounded on the need to study the child carefully for
b. Examine his study habits individualized instruction?
c. Talk the student and find out his problem � � ✔ a. Da Feltre � � ✔
d. Refer him to the guidance counselor b. Erasmus
118. Which is the TRUE foundation of the social order? c. Boccacio
a. Strong, political leadership d. Ascham
b. The reciprocation of rights and duties � � ✔ 124. Which of the following should a teacher do if she cannot pay the
c. Equitable distribution of wealth monthly installment of an appliance she got from a department store in
d. Obedient citizenry their town?
119. When do test, inventories and career information become a. Reject any notice of demand for payment to make the impression
effective for counseling services? that she did not receive
a. When the data generated are interpreted on time by professionally b. Move to another neighborhood to escape payment
competent person � � ✔ c. Inform the manager of the store personally and make a satisfactory
b. The psychological test result are still valid and reliable arrangement of payment on or before the due date of payment � �
c. When the records are updated ✔
d. When the records are kept for ready reference when needed d. Offer the return the used appliance to the store on the condition that
120. Which of the following is a major advantage in using arithmetic she will be refunded on the monthly installment she paid
mean? 125. Which of the following will you recommended to a senior high
a. It is more commonly used than other measures school scholar who is impregnated by a fellow student?
b. It discriminates between the lowest and the highest a. Tell her parent about her condition � � ✔
c. It is simple to compute � � ✔ b. Stop schooling till after she gives birth
d. It is more than stable than the median c. Direct her to an abortion clinic
d. Force her boyfriend to marry her
121. In preparing a multiple choice test how many options would be
126. The government prescribes a higher percentage on the
ideal?
administration of educational institution to Filipino citizens in order to
a. It is more commonly used than other measures
a. Minimize the unemployment problem
b. It discriminates between lowest and the highest
c. It is simple to compute b. Procedure globally competitive graduates � � ✔
d. It is more than stable than the median c. Protect the rights of the citizen
d. Ensure the teaching of Filipino
122. Learners often find it much easier to fit into a new social 127. Teacher should bear in mind that the period of greatest mental
situation when given encouragement and support. How can this be development is from:
done? a. 9 to 12 years
a. By giving him room responsibility
b. 12 to 15 years � � ✔
b. By assigning “peers or Buddies” to him � � ✔ c. 6 to 9 years
c. By giving him special help d. 3 to 6 years
d. By discovering his new interest 128. Which of the following is the best situation wherein you can
balance responsibility and accountability?
17
a. A teacher paid on an hour basis, takes her time with the subject b. Ex post facto
matter till end of period c. Descriptive
b. A teacher paid on an hour basis, teaches as much as she could for d. Historical
duration of the period � � ✔ 134. During the first grading period, a student obtained failing marks
c. A teacher paid on an hour basis, spends most of the time on the in five academic subjects. Which of the following tests would best
latest gossips in showbiz explain his performance?
d. A teacher paid on an hour basis, entertain her students with stories a. Mental ability
till the end of the period b. Personality
129. You have a pupil who is so talkative, naughty and aggressive that c. Attitude
he is a burden to the entire members of the class. How would you d. Aptitude � � ✔
remedy this problem? 135. Measuring the work done by gravitational force is a learning task.
a. Call the parents for dialogue At what level of cognition ?
c. Reprimand him always a. Application � � ✔
b. Report the case to the principal b. Knowledge
d. Talk to him seriously � � ✔ c. Evaluation
130. What should a teacher do before constructing items for a d. Comprehension
particular test? 136. Setting up criteria for scoring test is meant to increase their
a. Prepare a table specifications � � ✔ ________
b. Review the previous lessons a. Objectively **
c. Determine the length of time for answering it b. Reliability
d. Announce to students the scope of the test c. Validity
131. Under which of the multiple choice type of test can this question d. Usability
be classified? ‘ Which of the following statements expresses this
137. Which of the following you will do to an examinee you caught
concept in different forms?
cheating and who offered to a certain sum of money to keep quiet
a. Association � � ✔ a. Motion him to keep quiet and watch for him after the examination
b. Definition b. Confiscate his test paper and report him to the examination
c. Difference
supervisor � � ✔
d. Cause
c. Announce to all examinees the name of the cheater
132. Of the following types of test which is the most subjective in
d. Ignore him but let him feel you saw him
scoring?
138. Which of the following is the best situation wherein you can
a. Matching type
balance rights and authority?
b. Simple recall
a. Allow all their only daughter’s suitor to come and go as she pleases
c. Multiple choice
b. Censor all their only daughter’s suitor
d. Essay � � ✔ c. Choose a life-partner for their only daughter
133. In which of these research methods can the researcher control
d. Caution their only daughter’s choice of a boyfriend � � ✔
certain variable?
a. Experimental � � ✔
18
139. In testing which of the following is referred to as cultural bias? c. Educational and vocational aptitude
a. Test items are more familiar with some culture � � ✔ b. Quality and quantity of previous teaching
b. Some culture do better on test than others d. Capacity for future learning
c. Test will show who is more cultured 146. Which of the different types of test covers a wide variety of
d. Cultured people do better on tests objectives?
140. Which is the most obvious and familiar way of reporting a. True-false
variability? c. Matching
a. Range between highest with some culture b. Multiple choice � � ✔
c. Standard error of the mean d. Essay
b. Standard deviation � � ✔ 147. In a multiple choice test, keeping the options brief indicates
d. Distribution of raw scores ____________
141. The theory of identical elements in learning holds that transfer is a. Inclusion in the item irrelevant clues such as the use in the correct
facilitated when the ______ answer
a. Teacher uses different teaching devices b. Non inclusion of option that mean the same
b. Learner has a memory of specific responses c. Plausibility and attractiveness of the item � ✔
c. Development task is easily identifies d. Inclusion in the item any word that must otherwise repeated in each
d. Experience is similar to the application situation � � ✔ response
142. If this need is not met the adolescent tends to be critical and 148. Which of these criteria is the most important in test
always tries to find fault. This is the need constructions?
a. For recognition � � ✔ a. The stem should contain the central problem
c. to belong b. Items should be congruent with the objectives � � ✔
b. For adventure c. A table of specification should be prepared
d. for material security d. Options should be of almost the same length
143. The way a child talks, walks of manifest, gestures may have been
learned from models he had been exposed. This explains
1. It is a collection of religious poetry written by Rabindranath
a. Affective
Tagore.
b. Insight
a. Mahabharata
c. Social � � ✔ b. Gitanjali
d. Cognitive c. The Ramayana
144. Audio-visual aids are used in classroom teaching to __________ d. Bhagavad Gita
a. Help make learning more permanent Answer: B
c. Help clarify important concept 2. Robert Frost wrote the poem Acquainted with the Night
b. All of these � � ✔ from which the stanza is taken:
d. Arouse and sustain student’s interest I have been one acquainted with the night.
145. Which of the following is the MOST important purpose for using I have walked out in rain-back in rain.
achievement test? To measure the _________ I have out walked the farthest city light
a. Quality and quantity of previous learning � � ✔ The poet in the stanza talks of?
19
a. isolation and loneliness c. The Right One
b. joy getting out of the house d. The Pearl
c. youthful delight playing in the rain Answer: B
d. happiness in having been acquainted with the night 8. He was the American President who said, “Ask not what
Answer: A America will do for you, but what together we can do for the
3. Which is the BEST WAY to write the underlined portion of freedom of man.”
this sentence? Researchers also speculate that some teachers a. Gerald Ford
might have given boys more computer time because parents b. Franklin Roosevelt
and teachers expected boys to need computers for future c. Henry Truman
careers. d. John F. Kennedy
a. expected Answer: D
b. expecting 9. History is the witness that ______ passing of time.
c. will expect a. testifies
d. will have expected b. will testify
Answer: D c. testifies for
4. The wounded soldiers were visited by the president who d. testifies for the
honoured them with ____ for their _____. Answer: D
a. medals – valor 10. When I met Liza yesterday, it was the first time I _____ her
b. gun salute – bravery since Christmas.
c. appointments – dedication a. saw
d. money – sacrifice b. had seen
Answer: A c. have seen
5. Every June, Manila has its festival of outstanding Filipino d. have been seen
films. This was a project initiated by Answer: D
a. Arsenio Lacson 11. The commander ordered his men to hold on the fort. What
b. German Moreno was the message?
c. Antonio Villegas a. Surrender in arms
d. Ramon Bagatsing b. Keep on with the fight
Answer: C c. Disregard the peace negotiation
6. Which among the sounds below is voiceless? d. Rescue the hostage victims fast and early
a. /b/ Answer: B
b. /z/ 12. “She is a vision of feminine pulchritude.” This stands for
c. /g/ the following EXCEPT
d. /p/ a. Loveliness
Answer: D b. Comeliness
7. “Only the heart can see rightly.” This statement is lifted c. Homeliness
from what particular novel? d. Physical beauty
a. The Prince and the Pauper Answer: C
b. The Little Prince 20
13. The Nibelungenlied is a Answer: A
a. Latin Myth 19. The manager told his workers, “We have to reduce our
b. Chinese legend workforce.” What did he mean?
c. Russian folk song Workers are free to leave
d. Medieval German epic Workers are warned of possible lay off
Answer: D Workers have to double time on their jobs
14. An association wherein the name of something is Workers should submit themselves to a reducing gym
substituted by something that represents it. Answer: B
a. Metonymy 20. The parent remarked, “__________ I come late, just lock
b. Comparison the door.”
c. Euphemism a. In the absence
d. Personification b. In the process
Answer: A c. In the event
15. Because the moon rotates on its axis at the same time as it d. In the case
______ around the earth, we see the same side Answer: C
a. Revolve 21. The copyreader found the news story boring. He found it
b. Revolves full of _____.
c. Is revolving a. Adjectives
d. Has been revolving b. Verbs
Answer: B c. Pronouns
16. In English verse, a poetic foot having 1 stressed syllable d. Adverbs
followed by 1 unstressed syllable is ______. Answer: C
a. Trochaic 22. There were three guests on the stage. They were made up
b. Iambic of a parent, the governor and the principal. Who should be
c. Dactylic acknowledged first by the valedictorian?
d. Anapaestic a. The classmates
Answer: A b. The principal
17. Senators were accused by activists of washing their hands c. The governor
with the perfumes of Arabia. This state is commonly known as d. The parent
a. Guilt Answer: C
b. Triumph 23. What is suggested in the opening line? June 13, 1986 - they
c. Indecision came from all over America - 200,000 heroes strong, with their
d. Aggression families.
Answer: A a. The writer holds great admiration for the veterans
18. I suggest that he _____ in the room for one week. b. The writer is opposed to the Vietnam War
a. Stay c. The writer was a veteran of the war
b. Stayed d. The writer is a flag-waving patriot
c. Staying Answer: A
d. Stays 21
24. A readability mismatch happens when the reading levels of stanza:
books exceed the reading levels of the students. In this Drum on your drums, batter on your banjos, sob on the long
situation, the students experience frustration and they fall short cool winding saxophones. Go to it, O jazzmen.
of the expected or desired output. A student who finds himself/ Which words illustrate alliteration?
herself in such a mismatch will likely do which of the a. Batter and banjos
following? b. Sob and winding
a. Give an intelligent critique of the selection or story read c. Long and cool
b. Write a comprehensive reaction paper regarding the d. To and it
selection or story read Answer: A
c. Present an argument that the selection or story read was not 29. What is meant by AT SIXES AND SEVENS in this
properly written sentence?
d. Manifest an expected and commensurate emotional reaction We moved into the house last week, but I'm afraid everything,
to the selection or story read is still at sixes and sevens.
Answer: C a. The things have not been shipped.
25. What does this mean: Excuses are for losers… those who b. In a state of confusion
take responsibility for their actions are real winners in life. c. In an orderly manner
This tells of? d. The boxes are still intact
a. losers often fail because they find reasons for losing Answer: B
b. a winner can also be a quitter 30. What is the mood of these lines?
c. accountability of one’s action tells of bravery Daylight, I must wait for the sunrise
d. excuses are needed to justify any failure I must think of a new life
Answer: C And I mustn't give in.
26. A couple accepted a wedding invitation. They showed When the dawn comes
pleasure in these remarks tonight will be a memory, too
a. All guests congratulated the organizers and the couple And a new day will begin.
b. Guests came in and out of the ceremonies a. Afraid
c. The ceremonies were very impressive b. Sarcastic
d. The couple felt uneasy with the priest c. Depressed
Answer: C d. Hopeful
27. What correction should be made to this sentence? Answer: D
One of the theories is that the first child receives more of the Below is the LET Reviewer for General Education GENED:
parents' attention than other children so first-borns tend to be Filipino Part 1. We encourage readers/ reviewees to use the
more intellectual. comment boxes after the article for discussion.
a. Change is to are 1. Sinabi ni Carlos P. Romulo sa isa niyang akda, “Ang
b. Insert a comma after children Pilipino ay may dugong maharlika.” Ano ang kahulugan nito?
c. Change parents' to parent's a. Ang Pilipino ay nanggaling sa malayang lahi
d. Change theories to theory's b. Ang Pilipino ay sadyang mabuti ang budhi.
Answer: B c. Ang Pilipino ay galing sa mayamang lahi
28. Carl Sundburg wrote "Jazz Fantasia" which has for its first 22
d. Ang Pilipino ay madaling maipagbili c. Ipunin ang mga maykaya at ipalipad pauwi.
Answer: A d. Bayaang magsipag-uwian sa sariling sikap ang bawat isa
2. Alin sa mga sumusunod ang pinakatamang pangungusap? Answer: B
a. Ang mga kabataan sa lansangan ay naglalaro at 7. Ano ang pokus ng pandiwa sa pahayag na “Bumili ng
nagtatakbuhan sa lansangan. bagong sasakyan si Angelo”?
b. Ang kabataan ay naglalaro kung maliwanag ang buwan at a. Pokus sa direksyon
nagtatakbuhan sa lansangan. b. Pokus sa kagamitan
c. Ang mga kabataan ay naglalaro at nagtatakbuhan sa c. Pokus sa sanhi
lansangan kung maliwanag ang gabi. d. Pokus sa aktor
d. Ang mga kabataan kung maliwanag ang buwan ay Answer: D
nagtatakbuhan sa lansangan at naglalaro. 8. Kabaliwan at paglulustay ang inyong ginagawa taon-taon.
Answer: C Higit na marami ang maralitang nangangailangan ng salapi at
3. Piliin ang gawi ng pagsasalita: Kasiyahan ko nang makitang dunong. Ang nagsasalita ay
kayo’y nagmamahalan. a. Kuripot
a. Pangarap b. Matipid
b. Pagkontrol ng kilos c. Maramot
c. Pagkuha ng impormasyon d. Praktikal
d. Pagbabahagi ng damdamin Answer: D
Answer: D 9. Nasa anong kaganapan ng pandiwa ang pangungusap?
4. Ang kaugnayan ng pagkakapatay kina Burgos, Gomez at Naglaro ng basketball sa Rizal Stadium ang koponan ng aming
Zamora sa panitikang Pilipino ay pamantasan.
a. Nanatiling masigla ang diwang Pilipino a. Sanhi
b. Nakagising sa damdaming makabayan ng mga Pilipino b. Tagaganap
c. Natutong lumabag sa batas at lumaban sa may c. Kagamitan
kapangyarihan ang mga Pilipino d. Ganapan
d. Naimpluwensyahan ang diwang alipin ng mga Pilipino Answer: D
Answer: B 10, Sa aling salita magkakaroon ng saglit na paghinto kung
5. Laging UMUUKILKIL sa isipan ng ama ang nasirang pinagpipilitang si Rose ang nakabasag ng pinggan?
pangako ng anak. Hindi si Rose ang nakabasag ng pinggan.
a. Sumasagi a. Rose
b. Gumugulo b. Hindi
c. Bumubuhay c. Nakabasag
d. Sumasapi d. Pinggan
Answer: B Answer: B
6. Ang Kagawaran ng Ugnayang Panlabas ng Pilipinas ay 11. Anong tayutay ang tinutukoy sa pahayag.
gulong-gulo kapag may giyera sa ibang bansa. Ano ang unang Durog ang katawang bumagsak sa semento si Miguel.
ginagampanan ng ambassador ng bansa? a. Pagtutulad
a. Bilangin ang mga nasugatan at nasawi b. Pagbibigay katauhan
b. Alamin ang mga tirahan ng mga Pilipino sa bayang iyon. 23
c. Pagmamalabis b. Pangkayarian
d. Pagwawangis c. Pangnilalaman
Answer: C d. Palaugnayan
12. Sino ang pinagkalooban ng karangalan bilang “Unang Answer: C
Tunay na Makata” noong 1708? 18. Ang panukalang inihain niya ay lubhang malalim at
a. Jose dela Cruz mahirap arukin.
b. Felipe de Jesus a. Abutin
c. Francisco Balagtas b. Unawain
d. Jose Corazon de Jesus c. Sukatin
Answer: B d. Tanggalin
13. “Magtatrabaho ako at ikaw ay mag-aaral upang makatapos Answer: B
ka ng pag-aaral.” Anong uri ng pangungusap ito? 19. Ang wikang Filipino ay hawig sa mga wika sa Asya. Alin
a. Payak dito ang pinagmulan ng wikang Filipino?
b. Tambalan a. Bahasa
c. Hugnayan c. Nihonggo
d. Langkapan d. Mandarin
Answer: D d. Malayo-Polinesyo
14. Mag-aalas-singko na _____ umaga _____ magising siya. Answer: D
a. ng – ng 20. Ano ang katumbas ng “Dekalogo” ni Apolinario Mabini na
b. nang – nang nagsasaad ng aral sa Filipino?
c. ng – nang a. Mosaic Law
d. nang – kapag d. Code of Ethics ni Kalantiaw
Answer: C c. New Society ni Pres. Marcos
15. Ang butong tinangay ng aso, walang pagsalang nalawayan d. Code of Citizenship ni Pres. Quezon
ito. Ang kaisipang ito ay tumutukoy sa katotohanan ng ______. Answer: A
a. Pagnanakaw 21. Siya ay hinirang na taga-sensus ng bahay-bahay. Ano ang
b. Pagtatanan kanyang nalikom?
c. Pagpapakasal a. Ang bilang ng tao sa bahay
d. Pakikipagkaibigan b. Ang kayamanan ng may-bahay
Answer: B c. Ang datos tungkol sa mga bata sa bawat bahay
16. Anong uri ng pagbigkas ang salitang “dambuhala”? d. Ang datos tungkol sa mga naninirahan sa bawat bahay
a. Malumi Answer: D
b. Mabilis 22. Kami ang kabataang siyang magiging pag-asa ng bayan.
c. Maragsa Paano ginamit ang salitang may salungguhit?
d. Malumay a. Pagtukoy
Answer: A b. Pagpuri
17. Ang katawagan sa pangngalan, pang-abay, pang-uri at c. Panghalip
pandiwa ay? d. Pagmamalaki
a. Palabuuan 24
Answer: A a. Pinahihintutan ang pribadong paaralan na maglimbag sa
23. Walang tubig kahapon. Ito ay pangungusap na: wikang Ingles
a. May paksa b. Nilimbag sa Tagalog ang diploma sa di-Tagalog na bayan
b. Walang pandiwa c. Nilimbag sa Filipino ang diploma ngunit may Ingles
c. May panaguri d. Nalimbag sa Filipino ang diploma
d. Walang paksa Answer: D
Answer: D 29. Ang gintong panahon ng mga manunulat noong panahon
24. Ipinagmamalaki mo siya, BAHAG naman pala ang ng Amerikano ay batid sa uring
kanyang BUNTOT. Ang ibig sabihin ng salitang may malaking a. Sanaysay
titik ay: b. Nobela
a. Kuripot c. Panulaan
b. Traydor d. Maikling kwento
c. Duwag Answer: D
d. Mahiyain 30. Alin sa mga sumusunod ang may wastong gamit ng tinig
Answer: C ng pandiwa?
25. Sabihin ang gawi ng pananalitang ito: “Bawal tumawid, a. Ang hinog na papaya na kinuha sa puno ni Marie.
may namatay na dito!” b. Kinuha ni Marie ang hinog na papaya sa puno.
a. Pananakot c. Kinuha sa puno ang hinog na papaya ni Marie.
b. Pagtukoy d. Papayang hinog ang kinuha sa puno ni Marie.
c. Babala Answer: B
d. Paalala Below is the LET Reviewer for General Education GENED:
Answer: C Mathematics Part 1. We encourage readers/ reviewees to use
26. Pinakamahalagang nobelang Pilipino sa maraming taon na the comment boxes after the article for discussion.
nalimbag noong 1906 at tumalakay nang masinsinan sa 1. Two buses leave the same station at 8:00 pm. One bus
paksang puhunan laban paggawa at sa sosyalismo ang _____. travels north at the rate of 30 kph and the other travels east at
a. Luha ng Buwaya 40 kph. How many kilometers apart are the buses at 10 pm?
b. Banaag at Sikat a. 140 km
c. Ibong Mandaragit b. 100 km
d. Pangginggera c. 70 km
Answer: B d. 50 km
27. Ayon kay Balagtas, “ang laki sa layaw, karaniwa’y hubad” Answer: B
kaya ang mga bata ay 2. Calculate the mean absolute deviation of the following
a. jeproks numbers: 60, 80, 100, 75 and 95
b. nag-aartista a. 12.4
c. nakapagtatapos sa pag-aaral b. 14.2
d. hindi sumusunod sa magulang c. 16.1
Answer: D d. 18.9
28. Noong taong 1962, ano ang pagbabago sa paglimbag ng Answer: A
diploma at sertipiko ng pagtatapos? 25
3. Which of the following is the factorization of the binomial Answer: D
x2 - 42? 9. At what rate per annum should P2400 be invested so that it
a. (x + 4)(x + 2) will earn an interest of P800 in 8 years?
b. (x – 4)2 a. 6 ½ %
c. x(x + 2x + 2) b. 5 ½ %
d. (x – 4)(x + 4) c. 4.17 %
Answer: D d. 6 %
4. What value of x will satisfy the equation: 0.4(5x - 1470) = Answer: C
x? 10. The area of a rectangle is (x2 + 2x - 8). If its length is x +
a. 490 4, what is its width?
b. 2,130 a. x + 2
c. 1470 b. x - 2
Answer: D c. x + 1
5. Which of the following has the greatest value: d. x + 6
a. 3 + 32 + (3 + 3)2 Answer: B
b. 33 11. What is the value of 12⅙ - 3 ⅜ - 5 ⅔ + 20 ¾?
c. [(3 + 3)2]2 a. 21 1/8
d. (3 + 3 + 3)2 b. 22
Answer: C c. 23 7/8
6. The average of 5 different counting numbers is 20. What is d. 21
the highest possible value that one of the numbers can have? Answer: C
a. 20 12. The vertex angle of an isosceles triangle is 20°. What is the
b. 40 measure of one of the base angles?
c. 30 a. 150°
d. 90 b. 60°
Answer: D c. 75°
7. Three brothers inherited a cash amount of P62,000 and they d. 80°
divided it among themselves in the ratio of 5:4:1. How much Answer: D
more is the largest share than the smallest share? 13. Ana and Beth do a job together in three hours. Working
a. P75,000 alone, Ana does the job in 5 hours. How long will it take Beth
b. P30,000 to do the job alone?
c. P24,800 a. 3 and 1/3 hours
Answer: C b. 2 and 1/3 hours
8. What is the missing terms in the series 5, 20, 80, ___,1280, c. 3 hours
___, 20, 480? d. 7 and 1/2 hours
a. 50;210 Answer: D
b. 40;160 14. How much greater is the sum of the first 50 counting
c. 35;135 numbers greater than the sum of the first 100 counting
d. 320;5120 26
numbers? b. 74
a. 110 c. 88
b. 3,775 d. 85
c. 3,155 Answer: B
d. 1200 20. On a certain day, three computer technicians took turns in
Answer: N manning a 24-hour internet shop. The number of hours Cesar,
15. Which of the following has the largest value? Bert, and Danny were on duty was in the ratio 3:4:5,
a. 85 respectively. The shop owner pays them P50 per hour. How
b. 39 much would Danny receive for that day?
c. 65 a. P 230
d. 94 b. P500
Answer: A c. P160
16. A water tank contains 18 liters when it is 20% full. How d. P480
many liters does it contain when 50% full? Answer: B
a. 60 21. A retailer buys candies for P90.25. The pack has 35 pieces
b. 30 of candies. If she sells each candy for P2.25, how much profit
c. 58 does she make?
d. 45 a. P11.50
Answer: D b. P56.25
17. The edges of a rectangular solid have these measures: 1.5 c. P37.50
feet by 1½ feet by 3 inches. What is its volume in cubic d. P18.75
inches? 22. An online shop sells a certain calculator for P950 and
a. 324 charges P150 for shipping within Manila, regardless of the
b. 225 number of calculators ordered. Which of the following
c. 972 equations shows the total cost (y) of an order as a function
d. 27 of the number of calculators ordered (x)?
Answer: C a. y = (950 + 150)x
18. In a certain school, the ratio of boys to girls is 5 is to 7. If b. y = 150x +950
there are 180 boys and girls in the school, how many boys are c. x = 950y + 150
there? d. y = 950x + 150
a. 105 Answer: D
b. 90 23. One side of a 45° - 45° - 90° triangle measures x cm. What
c. 45 is the length of its hypotenuse?
d. 75 a. X √3 cm
Answer: D b. X cm
19. Ruben’s grades in 6 subjects are 88, 90, 97, 90, 91 and 86? c. (X √3)/2 cm
What is the least grade that he should aim for in the 7th subject d. X √2 cm
if he has to have an average of 88? Answer: D
a. 92 27
24. The legs of one right triangle are 9 and 12, while those of order not to be late for school?
another right triangle are 12 and 16. How much longer is the a. 6:42 am
perimeter of the larger triangle than the perimeter of the b. 6:27 am
smaller triangle? c. 6:57 am
a. 84 d. 7:02 am
b. 7 Answer: B
c. 12 30. Which common fraction is equivalent to 0.215?
d. 14 a. 43/200
Answer: C b. 27/125
25. Determine the midpoint of the line segment joining the c. 21/50
points (7, -3) and (-1, 6). d. 108/375
a. (2, 3/2) Answer: A
b. (2, -3/2) Below is the LET Reviewer for General Education GENED:
c. (3, 3/2) Science Part 1. We encourage readers/ reviewees to use the
d. (1, 5/2) comment boxes after the article for discussion.
Answer: C 1. Which of the following statements best describes a
26. Which of these has the longest perimeter? hypothetical element with an electron configuration of
a. A square 21 cm on a side 1s22s22p63s23p5?
b. A rectangle 19 cm long and 24 cm wide a. The hypothetical element has an atomic number of 11.
c. An equilateral triangle whose side is 28 cm b. The hypothetical element is a member of Group V,
d. A right triangle whose two legs are 24 and 32 cm otherwise called the Nitrogen Group.
Answer: D c. The hypothetical element is in the fifth position in the p-
27. How many square inches are in 2 square yard? block, along the third period of the periodic table.
a. 900 d. The hypothetical element is located at the third position of
b. 144 the p-block, along the fifth period of the periodic table.
c. 1296 Answer: C
d. 2,592 2. Despite the observed diversity among organisms, they are
Answer: D all made from the same set of biomolecules composed of
28. In a playground for Kindergarten kids, 18 children are monomeric units except:
riding tricycles or bicycles. If there are 43 wheels in all, how a. Proteins
many tricycles are there? b. Carbohydrates
a. 8 c. Nucleic Acids
b. 9 d. Lipids and Fats
c. 7 Answer: D
d. 11 3. What will be formed when radium isotope, with 88 protons
Answer: C and 138 neutrons undergoes alpha decay?
29. Nelia takes ¾ hour to dress and get ready for school. It a. Radon Atom (Rn222) with 86 Protons
takes 4/5 hour to reach the school. If her class starts promptly b. Francium Atom (Fr222) with 87 Protons
at 8:00 am; what is the latest time she can jump out of bed in 28
c. Actinium Atom (Ac222) with 89 Protons d. Ash from burnt rice hulls and straws are rich in compounds
d. Thorium Atom (Th232) with 90 Protons that could neutralize acidic soil so that more crops will grow
Answer: A 8. Acid rain occurs when ____________.
4. Which of the following examples best illustrates application Answer: D
of Boyle’s Law? 8. Acid rain occurs when _________.
a. A tire becomes harder as more air is pumped into it. a. carbon dioxide combines with water in the atmosphere.
b. A sealed aerosol can explodes when thrown into a fire. b. phosphorus-rich water in lakes evaporates to form
c. A balloon expands and bursts when exposed to direct phosphoric acid.
sunlight. c. sulfur released in burning fossil fuels combines with water
d. A scuba divers stops at certain depths as he ascends to the in the atmosphere.
ocean’s surface. d. excess hydrogen is released into the atmosphere to produce
Answer: D acids.
5. Three liquids A, B, C were studied in a laboratory. Liquid A Answer: C
was found to float over B and C. It was also found that liquid A 9. Why is it difficult to integrate nitrogen gas from the
flows fastest among the three. What can be said about liquid atmosphere into the nitrogen cycle of the biosphere?
A? a. Nitrogen is very abundant in the atmosphere
a. Densest and most viscous b. Living organisms quickly absorb nitrogen gas
b. Densest and least viscous c. Oceans quickly absorb nitrogen gas
c. Least dense and most viscous d. Few organisms can directly utilize atmospheric nitrogen
d. Least dense and least viscous Answer: D
Answer: D 10. Which of the following sentences about greenhouse effect
6. Which of the following statements represents a physical is INCORRECT?
change? a. Greenhouse gases trap heat in the atmosphere which are
a. An antacid tablet forms bubbles when dissolved in water. returned to the earth’s surface.
b. A flashlight beam slowly gets dimmer and finally dies out b. Greenhouse effect is important in maintaining the
over time. temperature of the earth.
c. The lawn grows thicker every day because fertilizers were c. Greenhouse effect is due to gases that absorb the green
added into the soil. region of light from the sun.
d. Frozen mango juice melted when left standing at room d. Greenhouse effect increases the overall surface temperature
temperature for 30 minutes. of the earth.
Answer: D Answer: C
7. Which of the following best explains why farmers burn rice 11. When a gardener propagates a plant by taking cuttings, he
straw and hull during seasons of harvest? plants his cutting in a well-watered soil in a plant pot. What is
a. Burning rice hulls and straws produce compounds that act as the most likely reason why he may then cover the plant and pot
repellant for pests which may damage plantation. with a lightly perforated polythene bag?
b. The smoke produced by burning rice hulls and straws a. To reduce the water demand of the cutting.
stimulate growth and fruit bearing of trees. b. To decrease the rate of gaseous exchange by the plant.
c. Rice hulls and straws are burned so that more spaces will be c. To reduce the chance of attack by pests.
available for planting next set of crops. 29
d. To protect the plant from cold weather. a. Northeast and southwest
Answer: A b. Trade wind and easterlies
12. It is a common observation that mushrooms thrive few c. Southwest and northeast
days after lightning strikes. Which among the following d. Westerlies and easterlies
biogeochemical cycles is involved in this process? Answer: A
a. Nitrogen cycle 17. Fog is a cloud with its base at or very near the ground. The
b. Carbon-Oxygen cycle formation of fog generally occurs after the ground has lost heat
c. Phosphorus cycle by:
d. Sulfur cycle a. Evaporation
Answer: A b. Convection
13. Which of the following factors contribute to an increase in c. Conduction
human population? d. Radiation
I. Immigration II. Emigration Answer: D
III. Natality IV. Mortality 18. Why do we see the sun rise in the east?
a. III and IV a. The earth revolves eastward.
b. II and III b. The earth rotates from west to east.
c. I only c. We are located in the 20th meridian.
d. I and III d. On the globe, we are located in the east.
Answer: D Answer: B
14. Shown below is a simple food web in a grassy community. 19. If a voltage of 100 volts produces a current of 5 amperes in
The arrow symbol means eaten by. What would happen if all an electrical device, what is the resistance?
snakes are killed? a. 95 Ohms
Grass>Grasshopper>Frog>Snake b. 20 Ohms
Bird>Snake c. 105 Ohms
a. Grass population would increase. d. 500 Ohms
b. Grasshopper population would increase. Answer: B
c. Bird and frog populations would increase. 20. Which of the following best differentiates an earthquake's
d. Grasshopper and bird populations would increase. intensity from its magnitude?
Answer: C a. Intensity describes 'the depth from which the earthquake
15. Which of the following shows mechanical weathering of originated’ while magnitude refers to ‘the energy of the
rocks? earthquake’.
a. formation of caverns b. Intensity cannot be measured while magnitude can be
b. acids dissolves rocks measured using a seismograph.
c. freezing water between rock particles c. Intensity refers to the strength of the quake while magnitude
d. iron in rocks combine with oxygen refers to the degree of destruction it caused at the epicenter.
Answer: C d. Intensity is a measure of how much damage an earthquake
16. PAGASA announces the approach of the seasonal winds. cause at the surface while magnitude is the strength of the
The familiar names used are Amihan and Habagat, quake.
internationally known as ________ and ________ respectively. 30
Answer: D lightning?
21. Comparing the speed of sound in liquids, gases, and solids, a. Light appears brighter in the sky.
the speed of sound is usually lowest in ____ and highest in b. Light travels faster than sound.
____. c. Sound travels 1.331 m/s
a. solids, gases d. Sound is released later actually
b. gases, liquids Answer: B
c. liquids, solids 27. Water has a higher specific heat than iron. What does this
d. gases, solids mean?
Answer: D a. Water is hotter than iron
22. Which has a greater density, a lake full of water or a cupful b. Water heats more rapidly than iron
of water? c. Water is more dense than iron
a. The cup full of water d. Water heats more slowly than iron
b. The lake full of water Answer: D
c. Not enough information 28. If a colorblind man marries a woman who has normal
d. They have the same density vision and no history of the disease, it is most probable that all
Answer: D of their :
23. A stainless steel spoon feels colder than a plastic spoon a. daughters will be carriers
because stainless steel b. daughters will be colorblind
a. absorbs less heat from the hand than plastic does c. sons will be carriers
b. is really colder than plastic d. sons will be colorblind
c. has a lower temperature than plastic Answer: A
d. conducts heat away from the hand faster than plastic does 29. Albino corn seedlings may grow several inches tall.
Answer: D However, they will eventually die, primarily because
24. Why is it NOT advisable to repeatedly open the door of a a. direct sunlight will destroy their cells
refrigerator? b. they lack adequate root system
a. It will loosen the hinges of the refrigerator's door c. they cannot produce their own food
b. Leads to wastage in electrical energy. d. they cannot obtain carbon dioxide
c. Repeated opening introduces bacteria in to the refrigerator. Answer: C
d. The warm air outside lowers the temperature inside thus 30. Mimosa pudica, locally known as Makahiya is called as
making the refrigeration less-efficient. such because its leaflets tend to close when touched. This
Answer: B organismal response to a given stimuli is called:
25. What kind of mirror is used in cars to give the driver a a. Hydrotropism
wider area and smaller image of the traffic behind him/her? b.Thigmotropism
a. Double concave c. Geotropism
b. Convex d. Phototropism
c. Plane Answer: B
d. Concave Below is the LET Reviewer for General Education GENED:
Answer: B Social Science Part 1. We encourage readers/ reviewees to use
26. Why do we hear thunder some seconds after seeing 31
the comment boxes after the article for discussion. d. Northeast
1. What factor was a major cause of both World War I and Answer: C
World War II? 6. The important factors which have contributed to the
a. The spread of Marxian ideas into Europe weakness in the internalization of desirable values is the?
b. The dropping of atomic bombs a. Lack of models among the very people expected to
c. Nationalism and national borders exemplify these values
d. The rise of totalitarian fascist states b. Use of approaches which are mainly cognitive rather than
Answer: D effective
2. In which organization is the Philippines a member to fight c. Lack of follow up systems from one grade level to another
communist aggression? d. Minimum recognition and appreciation given to teachers
a. APEC Answer: A
b. ASA 7. To govern is to rule and the government rules by laws.
c. UN Whose main duty is the enforcement of laws?
d. SEATO a. Police department
Answer: D b. Judiciary department
3. The theory that population increases by geometrical ratio c. Legislative department
while the means of subsistence increases by arithmetical ratio d. Executive department
is attributed to? Answer: D
a. Karl Marx 8. The term that refers to the class of Filipinos who were free
b. Robert Malthus and independent
c. Emile Durkheim a. Timawa
d. Aristotle b. Maharlika
Answer: B c. Aliping namamahay
4. In what instance is the Filipino double-standard morality d. Aliping saguiguilid
shown? Answer: A
a. A couple brings their sick child to the doctor then later to the 9. The Spanish expedition responsible for naming the
espiritista archipelago Filipinas
b. Young parents bring up their children in a manner different a. Magellan’s expedition
from how they were brought up b. Loarca expedition
c. Illiterate parents are eager to send their children to school c. Legaspi expedition
even if they themselves did not go to school d. Villalobos expedition
d. A married man who flirts with someone else other than his Answer: D
wife seems acceptable but a married woman who flirts with 10. The third and last military governor of the Philippines was
another man is condemned. a. Gen. Wesley Merritt
Answer: D b. Gen. Elwell Otis
5. Which part of Asia does the Arabian peninsula occupy? c. Gen. Arthur MacArthur
a. Northwest d. Gen. Douglas MacArthur
b. Southeast Answer: C
c. Southwest 32
11. Which Katipunan member commuted from Cavite to Answer: C
Manila to buy materials used to make ammunitions? 17. Which led to the creation of Pakistan as a nation in 1947?
a. Teresa Magbanua Religious differences between
b. Agueda Esteban a. Hindus and Christians
c. Teodora Alonso b. Christians and Muslims
d. Trinidad Tecson c. Hindus and Buddhists
Answer: B d. Hindus and Muslims
12.Which economic system is based on free enterprise? Answer: D
a. Globalism 18. Which is common to Sun Yat Sen and Mahatma Gandhi?
b. Mixed economies a. Promoted a society ruled by religious leaders
c. Capitalism b. Rejected violence as a way to political power
d. Communism c. Led a successful nationalistic movement in their respective
Answer: C countries
13. How is the so-called colonial mentality manifested? d. Supported Marxist philosophy to change existing
a. Cultural relativism governments
b. Cultural diversity Answer: B
c. Xenocentrism 19. Nebuchadnezzar was to the Babylonian Empire as Asoka
d. Ethnocentrism was to the ______ Empire.
Answer: C a. Roman
14. Which is a safeguard against unfair trade practices like b. Gupta
short-weighing? c. Greek
a. Total Quality Movement d. Maurya
b. Consumerism Answer: D
c. Consumer vigilance 20. With the opposition of the parity rights in mind, who does
d. Substandardization NOT belong to the group?
Answer: C a. Claro M. Recto
15. If the seven continents were arranged from largest to b. Jose Laurel
smallest, in which order does Australia fall? c. Manuel Roxas
a. 4th d. Pedro Taruc
b. 5th Answer: C
c. 6th 21. When the Filipino reformists asked for the assimilation of
8. 7th the Philippines by Spain, what did they ask for? For the
Answer: D Philippines to ____
16. In which continent can we find stormy Cape Horn which is a. Become independent from Spain
known as the graveyard of ships and sailors? b. Become a province of Spain
a. Africa c. Be independent from Spain with certain conditions
b. Australia d. Be represented in the Spanish Cortes
c. South America Answer: B
d. Asia 33
22. Who among the Presidents changed the date of our b. Eastern
celebration of Independence day from July 4 to June 12? c. Eastern and Central
a. Ramon Magsaysay d. Central and Western
b. Diosdado Macapagal Answer: B
c. Carlos Garcia 28. Which follows Pres. Garcia’s “Filipino First Policy”?
d. Ferdinand Marcos Filipinos
Answer: B a. Should buy and consume Filipino products only
23. In which country did the Philippines participate in the b. Should not contribute to the brain drain problem
world’s peacekeeping operations by sending doctors, nurses, c. Should be selective in the entry in the entry of foreign
soldiers and police? professionals in the country
a. Israel d. Were to be given first preference in all matters related to the
b. East Timor economic development of the country
c. Iraq Answer: D
d. Iran 29. Aside from the Philippines, which countries claim part of
Answer: C the Spratly islands in the South China Sea?
24. In which poem did Rizal write about offering one’s life for a. China, Thailand, Brunei, Malaysia
one’s country? b. China, Taiwan, Vietnam, Malaysia
a. A La Juventud Filipina c. Indonesia, China, Taiwan, Malaysia
b. Song of Maria Clara d. China, Thailand, Brunei, Malaysia
c. Sa Aking mga Kabata Answer: B
d. Mi Ultimo Adios 30. The following are our constitutional rights EXCEPT to
Answer: D a. Free access to legal assistance which shall not be denied due
25. As an effect of our geography, in which of the following to poverty
island/s do people travel mostly by water? b. Have access to all records of the government
a. Luzon c. Be presumed innocent until proven guilty
b. Mindanao d. Form association and labor unions
c. Visayas Answer: B
d. Visayas and Mindanao Below is the LET Reviewer for General Education GENED:
Answer: C Information and Communication Technology (ICT) Part 1. We
26. As an insular country, to which principle does the encourage readers/ reviewees to use the comment boxes after
Philippines adhere when it comes to territorial boundary? the article for discussion.
a. Two hundred nautical miles of the country’s coast 1. Which of the following is not a benefit of technology in
b. Three hundred fifty nautical miles from shore education?
c. Three-mile territorial limit a. Improves critical-thinking abilities.
d. Archipelagic doctrine b. Unlimited access to games and other internet sites.
Answer: D c. Allows cooperative learning.
27. Which part/s of the Visayas has/have comparatively more d. Increases self-expression.
excessively moist climate and limited arable lands? Answer: B
a. Western 34
2. Which situation shows that technology can be used to Answer: D
motivate students to learn? 7. He is considered to be the “Father of Computing” because
a. Teacher Eric employs educational games related to the of his contributions to the basic design of computer.
lesson they are taking. a. John Napier
b. Miss Cleo designs lessons which use cooperative learning b. William Oughtred
with technology integration. c. Blaise Pascal
c. Mr. Jimmy utilizes computer assisted instruction programs d. Charles Babbage
so that students will learn at their own pace. Answer: D
d. Teacher Sarah assigns projects to her class where self- 8. Which is the standard input device that accepts letters,
expression and creativity is acknowledged. numbers and commands from the user?
Answer: A a. Trackpad
3. He is considered as the “Father of Modern Media in b. Lightpen
Education”. c. Mouse
a. Jean Piaget d. Keyboard
b. Robert Gagne Answer: D
c. B.F. Skinner 9. Mr. Rico carefully studies the materials he acquired from the
d. Edgar Dale internet. He always examines if the author is qualified to
Answer: D present the material. This practice is?
4. It is a term to denote a whole range of technologies a. necessary to ensure that the materials are reliable.
associated with processing information and with sending and b. unethical because he does not trust the author.
receiving messages. c. unnecessary because all resources from the internet are
a. Educational Technology dependable.
b. Information and Communication Technology d. just a waste of time.
c. Media Technology Answer: A
d. Instructional Systems Design 10. When a teacher asks the consistency of a material taken
Answer: B from the internet with other available materials, he/she is
5. A methodology widely used for developing new training concerned with its?
programs. a. relevance
a. Microsoft b. accuracy
b. World Wide Web c. coverage
c. Instructional Systems Design d. currency
d. Computer Technology Answer: B
Answer: C
6. This is considered to be the first manual data processing
device developed in China in the 12th century A.D. PROFESSIONAL EDUCATION
a. Hieroglyphics 50 Items with Answer
b. Papyrus 1. Which of the following would be the most fitting action of a
c. Printing Press teacher who is having a relationship with his/her student?
d. Abacus 35
a. Deny the relationship. and principles.
b. Enjoy the relationship while it lasts. c. Requiring the learners the full mastery of the lessons.
c. Defer the relationship until they are ready to admit it. d. Encourage the learners to memorize factual knowledge.
d. Continue the relationship and exercise utmost professional 7) In line with the philosophy of Reconstructivism, which of the
discretion about this. following should be given emphasis in teaching?
2. Ms. de Leon is a sickly teacher. She gets to school even late and a. To seek for a better position in the society.
not feeling well. What provision in R.A. 7836 does she violates? b. To compare oneself with the less fortunate.
a. Teachers must be devoted, honest and punctual in performing c. To become economically self-reliant.
their duty. d. To designate one's superiority over the others.
b. Teachers should be physical, mentally and morally fit to teach. 8) One of the responsibilities of school administrators and faculty
c. Teachers should manifest genuine enthusiasm and pride in members is to communicate and cooperate harmoniously with the
teaching as a noble profession. community. Which of the following attributes best applies to this
d. All of the above responsibility?
3. "Education is a continuous process of experiencing and of a. Interfacing
visiting or reorganizing experiences" according to a Progressivist. b. Alignment
What does it mean? c. Partnership
a. Education begins and ends in school. d. Kinship
b. Education takes place anytime and anywhere. 9) This principle was violated when Teacher Rey talked badly
c. Education happens formally and informally. about the new school principal in front of his students by telling
d. Education goes on throughout life. them that the new principal is incompetent and arrogant.
4. If you have a foreign visitor, what would be the best gift to buy? a. Respect for authority
a. Imported antiques b. Fidelity
b. Expensive Buddha figurines c. Loyalty
c. Porcelain wares d. Confidentiality
d. Embroidered products from Quezon province 10) Which of the following is an integral part of the teaching
5. Which of the following would manifest best the Teacher-Student process?
Relationship? a. Teacher and principal
a. Teacher to inflict corporal punishment to offending student upon b. Lesson plan
the approval of parents. c. Classroom management
b. Teacher to inflict corporal punishment on offending student d. Rubrics
secretly. 11) Which of the following is the Fundamental Moral Principle?
c. Teacher to inflict corporal punishment to offending student at all a. Stealing is wrong.
times. b. Health is wealth.
d. Teacher to inflict no corporal punishment on offending student c. Do good and avoid evil.
at all times. d. Cheating is immoral.
6) Which of the following is a characteristic of a Pragmatist 12) "Moral example has greater effect on pupils' discipline than
Teacher? laws and punishment" is an advice for teachers from ________.
a. Equipping the learners with the basic abilities and skills. a. Confucius
b. Providing the learners with the opportunities to apply theories 36
b. Mohammed c. Operant conditioning
c. Lao Tzu d. Imitation
d. Plato 19) A teacher rewards a child for doing things correctly. This
13) According to Max Scheler's Hierarchy of Values, ________ is technique is called:
the highest form of values. a. Conditioning
a. Pleasure values b. Fading
b. Vital values c. Chaining
c. Spiritual values d. Reinforcement
d. Values of the Holy. 20) A person who is friendly and has a capacity to make people
14) Which of the following prepositions is attributed to Plato? laugh possesses:
a. Truth is relative to a particular time and place. a. Naturalistic intelligence
b. Human beings create their own truth. b. Spatial intelligence
c. Learning is the discovery of truth as latent ideas are brought to c. Intrapersonal intelligence
consciousness. d. Interpersonal intelligence
d. Sense perception is the most accurate guide to knowledge. 21) A student with Attention Deficit Disorder exhibits:
15) If reality is politically socially and economically formed, this a. Care for his/her personal things
philosophy is being followed: b. Impatient while waiting for his/her turn during games
a. Essentialism c. Completes work before shifting to another
b. Existentialism d. Excessively quiet
c. Progressivism 22) A child treats his friends highly aggressive. The reason behind
d. Pragmatism this attitude is his past experiences with his father who is also
16) In Piaget's concrete operational stage, a teacher should provide highly aggressive. This is demonstrated on what theory?
________. a. Social cognitive theory
a. Activities for hypothesis formulation. b. Cognitive developmental theory
b. Learning activities that involve problems of classification and c. Operant conditioning
order. d. Classical conditioning
c. Activities for evaluation purposes. 23) Based on Jung's psychological theory, a child who is shy and
d. Stimulating environment with ample objects to play with. prefers to be alone falls under what classification?
17) A grade 1 pupil likes to play with his friends, but gets angry a. Extrovert
when defeated. Piaget's theory states that this pupil is under what b. Ambivert
development stage? c. Paranoid
a. Concrete-operational stage d. Introvert
b. Sensorimotor stage 24) ________ puts emphasis on the importance of sensitive
c. Formal-operational stage periods in development.
d. Pre-operational stage a. Ecological theory
18) Fear of something that was caused by a painful experience in b. Social Cognitive theory
the past is an example of? c. Psychoanalytic theory
a. Insight d. Ethnological theory
b. Classical conditioning 37
25) Contrary to Freud's concept that the primary motivation of exemplifies ________.
human behavior is sexual in nature, Erikson's is ________ in a. Metacognition
nature. b. Insight learning
a. Social c. Analytical learning
b. Cultural d. Trial and error learning
c. Physical 31) A teacher attempting to develop a student's metacognitive
d. Biological skills teaches the student to:
26) ________ is described by Freud as the component/s of a. Recall past lesson
personality that is concerned with the idea of right and wrong. b. Visualize
a. Superego c. Formulate hypothesis
b. Superego and ego d. Think about their thinking
c. Id 32) An adolescent exhibits what common characteristics?
d. Ego a. Reasonable and secure
27) What idea about age is NOT right? b. Feels intense emotions and sense of disequilibrium
a. Chronological age is different from biological and psychological c. Slow but steady physical secure
age. d. Passive and obedient
b. Chronological age, biological age, psychological age and social 33) Instincts are under what mental dimension?
age are related. a. Ego and Superego
c. Psychological age and social age are two different things. b. Ego
d. Chronological age is the same as biological and psychological c. Id
age. d. Superego
28) ________ reflects the teacher's understanding of development 34) A child cannot do ________, according to Piaget's concrete
as results of maturation and learning. operational stage.
a. Patience when dealing with the slower ones. a. Doing mentally what was just physically done.
b. Creativity with classroom strategies or task. b. Reasoning applied to specific example.
c. Fairness when giving grades or school marks. c. Classifying objects into different sets.
d. Cheerfulness and enthusiasm when discussing. d. Imagining the steps necessary to complete an algebraic
29) ________ is Erikson's, Piaget's and Freud's thought about play. expressions.
a. Contribute to the child's mastery of his physical and social 35) What should teachers teach students to eradicate bullying
environment. among kids in school?
b. Makes a child's life enjoyable that he will tend to hate school a. Full development of talents
life later. b. Athletic skills
c. Prepares a child for an excellent academic performance in c. Respect for the dignity of persons
formal schooling. d. Full development of physical power
d. Develops the child's highly competitive attitude because of the 36) Education is the acquisition of the art and the utilization of
nature of play. knowledge. This statement means:
30) A student has been staring at a puzzle. He is figuring out how a. A learner's application of what she has learned is necessary.
to solve it and suddenly, an idea flashed in the student's mind and b. A learner's interest in art is commendable.
excitedly he was able to learn how to solve the puzzle. This 38
c. A learner's acquisition of information is sufficient. d. Metacognition
d. A learner's acquisition of information is not important. 43) A child received a candy after correctly completing his task.
37) Student's initiative is stifles by: The child always tries to complete all tasks correctly for him to
a. Rationalism have a candy once again. What is being shown in the situation?
b. Extreme authoritarianism a. Associative learning
c. "Utang na loob" b. Classical conditioning
d. "Bahala na" c. Operant conditioning
38) A piece of music may sound sad, but when each note is played, d. Pavlovian conditioning
there is nothing sad about it. This is based on the doctrine that 44) A child learns by association and by insights. This states that
says: the association and cognitive theories of learning are:
a. The whole experience is equal to the sum of its parts. a. Diametrically opposed
b. The whole experience is more than the sum of its parts. b. Complementary
c. The whole experience is less than the sum of its parts. c. Partly wrong
d. The whole experience is not in any way related to the sum of its d. Partly correct
parts. 45) Emotion's role on success and happiness is highlighted by
39) Which among the words below does NOT prevent the Daniel Goleman's theory on emotional intelligence. How can the
emergence of truth which the learners and teachers are in search teacher best show empathy in the case of fighting students?
of? a. Reprimand the students so that other will not follow the
a. Mindset misbehavior.
b. Mask b. Tell the students to stop fighting so that there will be peace in
c. Open mind the classroom.
d. Defense c. Make them realize how fighting negatively affects themselves
40) What a teacher should do if students are misbehaving in class? and others.
a. Send the misbehaving pupils to the guidance counselor. d. Establish roles and responsibilities to avoid arguments among
b. Involve the whole class in setting rules of conduct in the whole them.
class. 46) The television program BATIBOT was developed because:
c. Make a report to the parents about their children's misbehavior. a. Children learn by conditioning.
d. Set the rules for the class to observe. b. Children learn by discovery.
41) What is an inappropriate method in teaching young children? c. Children learn by trial and error.
a. Individual difference are expected and accepted d. Children learn by observing and imitating.
b. Integrated teaching-learning 47) Comprehension skills comprises the following, EXCEPT for:
c. Isolated skill development a. Finding the main idea
d. Positive guidance techniques b. Recreational reading
42) If a teacher believes that a child's mind is TABULA RASA, c. Sequencing events chronologically
then the teacher will most likely engage the students in ________ d. Reading critically
process for them to learn. 48) Who was responsible for stating that modeling is essential in
a. Sensory impressions pedagogy?
b. Reasoning a. Bandura
c. Reflections 39
b. Bruner
c. Skinner PROFESSIONAL EDUCATION (Secondary)
d. Thorndlike
49) To make her students participate more often during class MULTIPLE CHOICE
discussions, teacher Chai believes that she can apply Operant
Conditioning by way of: 1. Which does Naom Chomsky assert about language learning for
a. Giving more incentives to her students, such as additional children?
grades and small tokens. I. Young children learn and apply grammatical rules and
b. Emphasizing group sharing and guiding them in thinking vocabulary as they are exposed to them.
critically and innovatively. II. Begin formal teaching of grammatical rules to children
c. Instilling metacognition in them through modeling. as early as possible.
d. Letting them reflect on how they think about their subjects. III. Do not require initial formal language teaching for
50) "Porma" over substance is evident in students in one of these children.
instances.
a. Dolly submitted his comprehensive written report to Mrs. A. I and III C. I only
Topacio even if it was slightly soiled with tomato ketchup. B. II only D. I and II
b. Harvey submitted his written report on time to Mrs. Francisco
even if it reeked of the cigarette she smoked before class. 2. Which is/are the sources of man’s intellectual drives, according
c. Joel submitted his written report to Mrs. Cantiveros a week in to Freud
advance. A. Id C. Id -ego
d. Olivia submitted her poorly written report, which was printed B. Super ego D. Ego
on perfumed stationery to her English teacher, Mrs. Estrada.
3. Theft of school equipment like TV, computer, etc. by teenagers in
the community itself is becoming a common phenomenon. What
does this incident signify?
KEY TO CORRECTION A. Prevalence of poverty in the community.
1. D 2. B 3. D 4. D 5. D 6. B 7. C 8. C 9. B. Inability of school to hire security guards.
A C. Deprivation of Filipino schools.
10. C 11. C 12. A 13. D 14. C 15. C 16. B 17. D 18. D. Community’s lack of sense of co-ownership.
B
19. D 20. D 21. B 22. A 23. D 24. C 25. A 26. A 27. 4. As a teacher, you are reconstructionist, Which among these will
D be your guiding principle?
28. A 29. A 30. B 31. D 32. B 33. C 34. D 35. C 36. A. I must teach the child every knowledge, skills and value
A that he needs for a better future.
37. B 38. D 39. C 40. B 41. C 42. C 43. C 44. B 45. B. I must teach the child to develop his mental powers to the
C full.
46. D 47. B 48. A 49. A 50. A C. I must teach the child so he is assured of heaven.

40
D. I must teach the child that we can never have real B. Realist D. Existentialist
knowledge of anything.
11. Behavior followed by pleasant consequences will be
5. The concepts of Trust vs. mistrust, autonomy vs. shame & self- strengthened and will be more likely to occur in the future.
doubt, and initiative vs. guilt are most closely related with works Behavior followed by unpleasant consequences will be
of _____. weakened and will be more likely to be repeated in the future.
A. Erickson C. Freud Which one is explained
B. Piaget D. Jung A. Freud’s psychoanalytic theory
B. Thorndike’s law effect
6. Student Z does not study at all but when the Licensure C. B.F. Skinner’s Operant conditioning theory
Examination for Teachers comes, before he takes the LET, he D. D. Bandura’s social learning theory
spends one hour or more praying for a miracle, i.e., to pass the
examination. Which attitude towards religion or God is 12. Principal B tells her teachers that training in the humanities is
displayed? most important. To which educational philosophy does he
A. Religion as fake C. Religion as authentic adhere?
B. Religion as magic D. Religion as real A. Existentialism C. Progressivism
B. Perennialism D. Essentialism
7. Teacher B engages her students with information for thorough
understanding, for meaning and for competent application. 13. All subject in Philippine elementary and secondary schools are
Which principle governs Teacher B’s practice? expected to be taught using the integrated approach. This came
A. Constructivist C. Behaviorist about as a result of implementation of ___________.
B. Gestalt D. Cognitivist A. Program for Decentralized Education
B. School-Based Management
8. Student B claims: “I cannot see perfection but I long for it. So it C. Basic Education Curriculum
must be real. “under which group can he be classified? D. D. Schools First Initiative
A. Idealist C. Realist
B. Empiricist D. Pragmatist 14. Principal C shares this thought with the teachers: Subject matter
should help students understand and appreciate themselves as
9. What does extreme authoritarianism in the home reinforce in unique individual who accept complete responsibility for their
learners thoughts, feelings and actions. “ From which philosophy is this
A. Doing things on their own initiative thought based?
B. Ability to direct themselves A. Perennialism C. Existentialism
C. Dependence on others for directions. B. Essentialism D. Progressivism
D. Creativity in work.
15. Based on Piaget’s theory, what should a teacher provide for
10. You arrive at knowledge by re-thinking of latent ideas. From children in the concrete operational stage?
whom does this thought come? A. Activities for hypothesis formulation.
A. Experimentalist C. Idealist
41
B. Learning activities that involve problems of classification and 21. Availment of the Philippine Education Placement Test for adults
ordering. and out-of-school youths is in support of the government’s
C. Games and other physical activities to develop motor skills. educational program towards _____.
D. Stimulating environment with ample objectives to play with. A. equitable access C. quality and relevance
B. quality D. relevance
16. To come closer to the truth we need to “go back to the things
themselves.” This is the advice of the ______. 22. A mother gives his boy his favorite snack every time the boy
A. Behavioris C. idealist cleans up his room. Afterwards, the boy cleaned his room
B. phenomenologists D. pragmatists everyday in anticipation of the snack. Which theory is
illustrated?
17. Researches conducted show that teacher’s expectations of A. Associative learning
students become self-fulfilling prophecies. What is this B. Classical conditioning
phenomenon called? C. Operant conditioning
A. Halo effect C. Ripple effect D. Pavlonian conditioning
B. Pygmalion effect D. Hawthorne effect
23. Which group of philosophers maintain the “truth exists in an
18. Teacher F is convinced that whatever a student performs a objective order that is independent of the knower”?
desired behavior, provided reinforcement and soon the student A. Idealists C. Existentialists
will learn to perform the behavior on his own. On which B. Pragmatists D. Realists
principle is Teacher F’s conviction based?
A. Cognitivism C. Behaviorism 24. Under which program were students who were not
B. Environmentalism D. Constructivism accommodated in public elementary and secondary schools
because of lack of classroom, teachers, and instructional
19. Teacher U teaches to his pupils that pleasure is not the highest materials, were enrolled in private schools in their respective
good. Teacher’s teaching is against what philosophy? communities at the government’s expense?
A. Realism C. Epicureanism A. Government Assistance Program
B. Hedonism D. Empiricism B. Study Now-Pay later
C. Educational Service Contract System
20. Studies in the areas of neurosciences discloses that the human D. National Scholarship Program
brain has limitless capacity. What does this imply
A. Some pupils are admittedly not capable of learning. 25. Which of the following prepositions is attributed to Plato?
B. Every pupil has his own native ability and his learning is A. Truth is relative to a particular time and place.
limited to this native ability. B. Human beings create their own truths.
C. Every child is a potential genius. C. Learning is the discovery of truth as latent ideas are brought to
D. Pupil can possibly reach a point where they have learned consciousness
everything. D. Sense perception is the most accurate guide to knowledge.

42
26. In a treatment for alcoholism, Ramil was made to drink an
alcoholic beverage and then made to ingest a drug that produces 33. If teacher wants to test student’s ability to organize ideas, which
nausea. Eventually, h nauseated at he sight and smell of alcohol type of test should she formulate?
and stopped drinking alcohol. Which theory explains this? A. Short answer C. Essay
A. Operant conditioning C. Associative learning B. Technical problem type D. Multiple-choice type
B. Social learning theory D. Attribution theory
34. Who among the following needs less verbal counseling but needs
27. In a social studies class, Teacher I present a morally ambiguous more concrete operational forms of assistance? The child who
situation and asks his students what they would do. On whose ________.
theory is Teacher I’s technique based A. has mental retardation
A. Kohlberg C. Piaget B. has attention-deficit disorder
B. Bandura D. Bruner C. has learning disability
D. has conduct disorder
28. Quiz to formative test while periodic is to _________.
A. criterion-reference test C. norm-reference test 35. Which applies when skewness is zero?
B. summative test D. diagnostic test A. Mean is greater than median.
B. Median is greater than the mean.
29. Your teacher is of the opinion that the world and everything in it C. Scores have three modes.
are ever changing and so you the skill to cope with change. What D. Scores are normally distributed
is his governing philosophy?
A. Idealism C. Experimentalism 36. Which does NOT belong to the group of alternative learning
B. Existentialism D. Realism systems
A. Multi-grade grouping C. Graded education
30. The search for related literature by accessing several data bases B. Multi-age grouping D. Non-graded grouping
by the use of a telephone line to connect a computer that have
database is termed ______. 37. The first thing to do in constructing a periodic test is for a teacher
A. compact disc search C. on-line search to ______.
B. manual search D. computer search A. decide on the number of items for the test
31. In which competency do my students find the greater difficulty? B. go back to her instructional objectives
In item with a difficulty index of C. study the content
A. 0. 1 C. 0.5 D. decide on the type of test to construct
B. 0. 9 D. 1.0
38. Which one can enhance the comparability of grades?
32. We encounter people whose prayer goes like this: O God, if there A. Using common conversation table for translating test scores in
is a God; save my soul if I have a soul,” from whom is this to ratings.
prayer? B. Formulating tests that vary from one teacher to another.
A. Stoic C. Agnostic C. Allowing individual teachers to determine factors for rating
B. Empiricist D. Skeptic
43
D. Individual teachers giving weights to factors considered for 44. In the context on the theory on multiple intelligences, what is one
rating. weakness of the paper-pencil test?
A. It is not easy to administer.
39. The cultivation of reflective and meditative skills in teaching is an B. It puts the non-linguistically intelligent at a disadvantage
influence of _____. C. It utilizes so much time.
A. Shintoism C. Confucianism D. It lacks reliability.
39 B. Zen Buddhism D. Taoism
45. NSAT and NEAT results are interpreted against set mastery level.
40. Teacher Y does norm-referenced interpretation of scores. Which This means that NSAT and NEAT fall under _________.
of the following does she do? A. intelligence test C. criterion-referenced test
A. She describes group performance in relation to a level of B. aptitude test D. norm-referenced test
mastery set.
B. She uses a specified content as its frame of reference. 46. Teacher B uses the direct instruction strategy. Which sequence of
C. She compares every individual student’s scores with others’ steps will she follow?
scores. I. Independent practice
D. She describes what should be their performance. II. Feedback and correctiveness
III. Guided student practice
41. The best way for a guidance counselor to begin to develop study IV. Presenting and structuring
skills and habits in underachieving student would be to ______. V. Reviewing the previous day’s work
A. have these underachieving students observe the study habits of
excelling students. A. V-IV-III-II-I C. V-II-IV-III-I
B. encourage students to talk study habits from their own B. III-II-IV-I-V D. I-V-II-III-IV
experiences
C. have them view filmstrips about various study approaches 47. Which guideline must be observed in the use of prompting to
D. give out list of effective study approaches shape the correct performance of your students?
A. Use the list intrusive prompt first.
42. Teacher F wanted to teach the pupils the skills to do cross B. Use all prompts available.
stitching. He check up quiz was a written test on the steps of C. Use the most intrusive prompt first.
cross stitching. What characteristic of a good test does it lack? D. Refrain form using prompts.
A. Scorability C. Objectivity
B. Reliability D. Validity 48. What measure/s of central tendency do/es the number 16 represent
in the following data: 14,15,17,16,19,20,16,14,16?
43. For which may you use the direct instruction method? A.Mode C. Median
A. Become aware of the pollutants around us. B.Mode and median D. Mean
B. Appreciate Milton’s Paradise Lost.
C. Use a microscope properly. 49. Which holds true to standardized tests?
D. Distinguish war from aggression. A. They are used for comparative purposes.
B. They are administered differently.
44
C. They are scored according to different standards. 55. The burnout malady gets worse if a teacher doesn’t intervene to
D. They are used for assigned grades. change whatever areas he or she in control. Which one can renew
a teacher’s enthusiasm?
50. Study this group of tests which was administered with the A. Stick to job C. Judge someone else as wrong
following results, then answer the question. B. Initiate changes in jobs D. Engage in self-pity
Subject Mean SD Ronnel’s scrore
Math 56 10 43 56. If teacher has to ask more higher-order questions, he has to ask
Physics 41 9 31 more ____ questions.
English 80 16 109 A. closed C. concept
B. Fact D. convergent
In which subject(s) were the scores most homogenous?
A. Math C. Physics 57. What can be said of Peter who obtained a score of P75 in a
B. English D. Physics and Math Grammar objective test?
51. In the parlance of test construction what does TOS mean? A. His rating is 75 items in the test correctly.
A. Table of Specifics C. Table of Specific Test Items B. He answered 75 % of the test items correctly.
B. Term of Specifications D. Table of Specifications C. He answered 75.
D. He performed better that 5% of his classmate.
52. Which is a major advantage of a curriculum-based assessment?
A. It is informal nature. 58. I drew learners into several content areas and encouraged them to
B. It connects testing with teaching. solve a complex question for interdisciplinary teaching. Which
C. It tends to focus on anecdotal information on student progress. strategy did I use?
D. It is based on a norm-referenced measurement model. A. Problem-centered learning
B. Unit method
53. Teacher H gave first-grade class a page with a story in which C. Reading-writing activity
picture take the place of some words. Which method did she use? D. D. Thematic instruction
A. The whole language approach
B. The Spaulding method 59. Which guidance in test construction is NOT observed in this test
C. The rebus method item “ Jose Rizal wrote ____”.
D. The language experience approach A. The central problem should be packed in the stem.
B. There must be only one correct answer.
54. Out of 3 distracters in a multiple choice test items, namely B, C, C. Alternatives must grammatical parallelism.
and D, no pupil choice D as answer. This implies that D is D. The alternative must be plausible.
________.
A. an ineffective distracter 60. To elicit more students’ response, Teacher G. made use of covert
B. a vague distracter responses. Which one did she NOT do?
C. an effective distracter A. She had the students write their response privately.
D. a plausible distracter B. She showed the correct answers on the overhead after the
students have written their responses.
45
C. She had the students write their responses privately then called D. None of the above
each of them.
D. She refrained from judging on the student’s responses. 66. Teacher W wants to review and check on the lesson of the previous
day. Which one will be most reliable?
61. Direct instruction id a facts, rules, and actions as indirect A. Having students identify difficult homework problems.
instruction is for _____ ,____, _____. B. Having students correct each other’s work.
A. hypotheses, verified data and conclusions C. Sampling the understanding of a few students.
B. concepts, patterns and abstractions D. Explicitly reviewing the task-relevant information necessary
C. concepts, processes and generalizations for the day’s lesson.
D. guesses, data and conclusions
67. Shown a picture of children in sweaters inside the classrooms,
62. Which test has broad sampling of topics as strength? the students were asked this question: In what kind of climate do
A. Objective test C. Essay test these children live? This is a thought questions on ______.
B. Short answer test D. Problem test A. inferring C. applying
. B. creating D. predicting
63. Teacher T taught a lesson denoting ownership by means of
possessives. He first introduced the rule, then gave examples, 68. Study this group of tests which was administered with the
followed by class exercises, then back to the rule before he following results, then answer the question.
moved into second rule. Which presenting technique did he use Subject Mean SD Ronnel’s scrore
A. Combinatorial C. Part-whole Math 56 10 43
B. Comparative D. Sequential Physics 41 9 31
English 80 16 109
64. In his second item analysis, Teacher H found out that more from
the lower group got the test item # 6 correctly. This means that the In which subject(s) did Ronnel perform best in relation to the
test item _____. group’s performance?
A. has a negative discriminating power.
B. has a lower validity. A. Physics and Math C. Physics
C. has a positive discriminating power. B. English D. Math
D. has a high reability.
69. Which criterion should guide a teacher in the choice of
65. Teacher E discussed how electricity flows through wires and what instructional devices?
generates the electric charge. Then she gave the students wires, A. Attractiveness C. Novelty
bulbs, switches, and dry cells and told the class to create a circuit B. Cost D. Appropriateness
that will increase the brightness of each bulb. Which one best 70. Study this group of tests which was administered with the
describes the approach used? following results, then answer the question.
A. It used a taxonomy of basic thinking skills. Subject Mean SD Ronnel’s scrore
B. It was constructivist. Math 56 10 43
C. It helped students understand scientific methodology. Physics 41 9 31
46
English 80 16 109
77. Which is one role play in the pre-school and early childhood
In which subject(s) did Ronnel perform poorly in relation to years?
the group’s performance? A. Develops competitive spirit.
A. English C. Math B. Separates reality from fantasy.
B. English and Math D. Physics C. Increase imagination due to expanding knowledge and
emotional range.
71. “What is most likely to happen to our economy when export D. Develops the upper and lower limbs.
continuously surpasses import” is a thought question on ______.
A. creating C. relating cause-and-effect 78. Teacher A discovered that his pupils are very good in dramatizing.
B. synthesizing D. predicting Which tool must have helped him discover his pupils’ strength?
A. Portfolio assessment C. Journal entry
72. Which method has been proven to be effective in courses that B. Performance test D. Paper-and-pencil test
stress acquisition of knowledge?
A. Socratic method C. Indirect instruction 79. Teacher M’s pupils are quite weak academically and his lesson is
B. Cooperative learning D. Mastery learning already far behind time table. How should Teacher M proceed with
his lesson?
73. Which is the first step in planning an achievement test? A. Experientially C. Logically
A. Define the instructional objective. B. B. Inductively D. Deductively
B. Decide on the length of the test.
C. Select the type of test items to use. 81. I want to teach concepts, patterns and abstractions. Which method
D. Build a table of specification. is most appropriate?
A. Indirect instruction C. Direct instruction
74. Which activity should a teacher have more for his students if he B. Discovery D. Problem solving
wants to develop logical-mathematical thinking?
A. Drama C. Problem solving 82. If your Licensure Examination for Teacher (LET) items sample
B. Choral readingD. Storytelling adequately the competencies listed in the syllabi, it can be said
that LET possesses ______ validity.
75. Why should a teacher NOT use direct instruction all the time A. concurrent C. content
A. It requires much time. B. construct D. predictive
B. It requires use of many supplemental materials.
C. It is generally effective only in the teaching of concepts and 83. Read the following then answer the questions:
abstractions.
D. It reduces student’s engagement in learning. TEACHER: IN WHAT WAYS OTHER THAN THE PERIODIC
TABLE MIGHT WE PREDICT THE
76. Which are direct measures of competence? UNDISCOVERED ELEMENTS?
A. Personality tests C. Paper-and-pencil test
B. B. Performance tests D. Standardized tests
47
BOBBY: WE COULD GO TO THE MOON AND SEE IF 87. In self-directed learning, to what extent should a teacher’s
THERE ARE SOME ELEMENTS THERE WE “scaffolding” be?
DON’T HAVE. A. To a degree the student needs it.
BETTY: WE COULD DIG DOWN TO THE CENTER OF B. None, to force the student to learn by himself
THE EARTH AND SEE IF WE FIND ANY OF C. To the maximum, in order to extend to the student all the help
THE MISSING ELEMENTS. he needs.
RICKY: WE COULD STUDY DEBRIS FROM THE D. To the minimum, to spend up development of student’s sense
METEORITES-IF WE CAN FIND ANY. of independence.
TEACHER: THOSE ARE ALL GOOD ANSWERS. BUT
WHAT IF THOSE EXCURSIONS TO THE 88. Which is a form of direct instruction?
MOON, TO THE CENTER OF THE EARTH OR A. Discovery process C. Programmed instruction
TO FIND METEORITES WERE TOO COSTLY B. B. Problem solving D. Inductive reasoning
AND TIME CONSUMING? HOW MIGHT WE
USE THE ELEMENTS WE ALREADY HAVE 89. With synthesizing skills in mind, which has the highest diagnostic
HERE ON EARTH TO FIND SOME NEW ONES? value?
A. Essay test C. Completion tes
Question: The Teacher questions in the above exchange are examples B. Performance test D. Multiple choice test
of ____ questions. 90. How can you exhibit legitimate power on the first day of school?
A. Fact C. Direct A. By making your students feel they are accepted for who they
B. Concept D. Closed are.
B. By informing them you are allowed to act in loco parents.
84. In Krathwoh’s taxonomy of objectives in the affective, which is C. By making them realize the importance of good grades.
most authentic? D. By making them feel you have mastery of subject matter.
A. Characterization C. Responding
B. Valuing D. Organization 91. Based on Edgar Dale’s Cone of Experience, which activity is
farthest from the real thing?
85. “In the light of the facts presented, what is most likely to happen A. View images C. Watch a demo
when…?” is a simple thought question on _____. B. Attend exhibit D. Hear
A. inferring C. synthesizing
B. generalizing D. justifying 92. Which can effectively measure student’s awareness of values?
A. Projective techniques C. Likert scales
86. The teacher’s first task in the selection of media in teaching is to B. Moral dilemma D. Anecdotal record
determine the
A. choice of the students 93. I combined several subject areas in order to focus on a single
B. availability of the media concept for interdisciplinary teaching. Which strategy/method did
C. objectives of the lesson I use?
D. technique to be used A. Problem-entered learning
B. Thematic instruction
48
C. Reading–writing activity D. Give them work on the level of the other students and work a
D. Unit method little above the classmates level to challenge them.

94. The test item ”Group the following items according to shape” is a 100. With-it-ness, according to Kourin, is one of the characteristics of
thought test item on _______. an effective classroom manager. Which phrase goes with it?
A. creating C. classifying A. Have hands that write fast.
B. generating D. comparing B. Have eyes on the back of your hands.
C. Have a mouth ready to speak.
95. For maximum interaction, a teacher ought to avoid _____ D. Have minds packed with knowledge.
questions.
A. informational C. leading 101. On whose philosophy was A.S.Neil Summerhill, one of the most
B. rhetorical D. divergent experimental schools based?
A. A. Rousseau C. Montessori
96. By what name is Socratic method also known ? B. B. Pestalozzi D. John Locke
A. Mastery learning C. Morrison method
B. Indirect instruction D. Questioning Method 102. Which Filipino trait works against the shift in teacher’s role from
teacher as a fountain of information to teacher as facilitator?
97. Which is an appropriate way to make manage off-task behavior? A. Authoritativeness C. Hiya
A. Make eye contact. B. Authoritarianism D. Pakikisama
B. Stop your class activity to correct a child who is no longer on
task. 103. Teacher A is a teacher of English as Second Language: she uses
C. Move closer to the child. vocabulary cards, fill-in-the blank sentences, dictation and
D. Redirect a child’s attention to task and check his progress to writing exercises in teaching a lesson about grocery shopping.
make sure he is continuing to work Based on this information, which of the following is a valid
conclusion?
98. Which one can best evaluate student’s attitudinal development? A. The teacher is applying Bloom’s hierarchy of cognitive
A. Essay test C. Observation learning.
B. Portfolio D. Short answer test B. The teacher is teaching variety of ways because not all
students learn in the same manner.
99. What should a teacher do for students in his class who are on C. The teacher wants to make her teaching easier by having less
grade level talk.
A. Give them materials on their level and let them work at a pace D. The teacher is emphasizing reading and writing skills.
that is reasonable for them, trying to bring them up to a grade
level. 104. With specific details in mind, which one has (have) a stronger
B. Give them the same work as the other students, because they diagnostic value?
will absorb as much as they are capable of. A. Multiple choice test
C. Give them the same work as the other students, not much, so B. Non-restricted essay test
that they won’t feel embarrassed. C. Restricted essay test
49
D. Restricted and non-restricted essay tests
110.It is not wise to laugh at a two-year old child when he utters bad
105. Teacher B is a teacher of English as Second Language: she uses word because in his stage he is learning to __________.
vocabulary cards, fill-in-the blank sentences, dictation and A. considered other’s views
writing exercises in teaching a lesson about grocery shopping. B. distinguish right from wrong
Based on this information, which of the following is a valid C. socialize
conclusion? D. distinguish sex differences
A. The teacher is reinforcing learning by giving the same
information in a variety of methods. 111.Research tells that teachers ask mostly content questions. Which
B. The teacher applying Bloom’s hierarchy of cognitive learning. of the following terms does NOT refer to content questions?
C. The teacher is wants to do less talk. A. Closed C. Concept
D. The teacher is emphasizing listening and speaking skills. B. Direct D. Convergent

106. In a criterion-referenced testing, what must you do to ensure 112.In mastery learning, the definition of an acceptable standard of
that your test is fair performance is called a
A. Make all of the questions true or false. A. behavior C. SMART
B. Ask each student to contribute one question. B. condition D. criterion measure
C. Make twenty questions but ask the students to answer only ten
of their choice. 113. Based on Edgar Dale’s Cone of Experience, which activity is
D. Use objectives for the unit as guide in your test construction. farthest from the real thing?
A. Read C. View images
107. Read this question: “How will you present the layers of the earth B. Hear D. Attend exhibit
to your class?” This is a question that _________.
A. directs 114. After giving an input on a good paragraph, Teacher W asks her
B. leads the students to evaluate students to rate a given paragraph along the elements of a good
C. assesses cognition paragraph. The students’ task is in level of _________.
D. probes creative thinking A. application C. evaluation
B. analysis D. synthesis
108. In Krathwoh’s affective domain of objectives, which of the
following is the lowest level of effective behavior? 115. Which is one characteristic of an effective classroom
A. Valuing C. Responding management?
B. Characterization D. Organization A. It quickly and unobtrusively redirects misbehavior once it
occurs.
109. Which is NOT a sound purpose for asking questions? B. It teaches dependence on other for self-control.
A. To probe deeper after an answer is given C. It respects cultural norms of a limited group students.
B. To discipline a bully in class D. Strategies are simple enough to be used consistently
C. To remind students of a procedure
D. To encourage self-reflection 116. Read the following then answer the questions:
50
TEACHER: IN WHAT WAYS OTHER THAN THE PERIODIC orally material at the fourth or fifth grade level. The most probable
TABLE MIGHT WE PREDICT THE cause/s of this student’s reading problem is/are ______.
UNDISCOVERED ELEMENTS? A. emotional factors
BOBBY: WE COULD GO TO THE MOON AND SEE IF B. poor teaching
THERE ARE SOME ELEMENTS THERE WE C. neurological factors
DON’T HAVE. D. immaturity
BETTY: WE COULD DIG DOWN TO THE CENTER OF THE
EARTH AND SEE IF WE FIND ANY OF THE 119. Which questioning practice promotes more class interaction?
MISSING ELEMENTS. A. Asking the question before calling a student
RICKY: WE COULD STUDY DEBRIS FROM THE B. Focusing on divergent questions.
METEORITES- IF WE CAN FIND ANY. C. Focusing on convergent questions.
TEACHER: THOSE ARE ALL GOOD ANSWERS BUT D. Asking rhetorical questions
WHAT IF THOSE EXCURSIONS TO THE MOON,
TO THE CENTER OF THE EARTH OR TO FIND 120. The following are sound specific purpose of questions EXCEPTS
METEORITES WERE TOO COSTLY AND TIME A. To call the attention of an inattentive student
CONSUMING? HOW MIGHT WE USE THE B. To teach via student answers
ELEMENTS WE ALREADY HAVE HERE ON C. To stimulate learners to ask questions
EARTH TO FIND SOME NEW ONES? D. To arouse interest and curiosity

Question: which questioning strategy/ies does/do the exchange 121. Which technique should a teacher use to encourage response if
of thoughts above illustrate? his students do not respond to his question?
A. Funneling A. Ask a specific student to respond, state the question, and wait a
B. Sowing and reaping response.
C. Nose-dive B. Tell the class that it will have detention unless answers are
D. Extending and lifting forthcoming.
C. Ask another question, an easier one.
117. How can you exhibit referent power on the first day of school? D. Wait for a response.
A. By making them feel you know what you are talking about.
B. By telling them the importance of good grades. 122. Which types of play is most characteristic of a four to six-year
C. By reminding your students your authority over them again old child?
and again. A. Solidarity plays and onlookers plays
D. By giving your students a sense of belonging and acceptance. B. Associative and cooperative plays
C. Associative and onlookers plays
118. A sixth grade twelve-year old boy comes from a dysfunctional D. Cooperative and solidarity plays
family and has been abused and neglected. He has been to two
orphanages and three different elementary schools. The student 123. The principle of the individual difference requires teachers to
can decode on the second grade level, but he can comprehend ___________.
A. give greater attention to gifted learners
51
B. provide for a variety of learning activities 129. John Watson said “Men are built not born. ” What does this
C. treat all learners alike while in the classroom statement point to?
D. prepare modules for slow learners in class A. The ineffectiveness of training on a person’s development.
124. Referring to Teacher S, Nicolle describes her teacher as “fair, B. The effect of environmental stimulation on a person’s
caring and someone you can talk to”. Which power or leadership does development.
Teacher S have? C. The absence of genetic influence on a person’s development.
A. Referent power C. Reward power D. The effect of heredity.
B. Legitimate power D. Expert power
130. Teacher H strives to draw participation of every student into
125. During the Spanish period, what was/were the medium/media of classroom discussion. Which student’s need is she trying to
instruction in schools? address? The need
A. The Vernacular C. Spanish A. to show their oral abilities to the rest of the class
B. English D. Spanish and the Vernacular B. to be creative
C. to feel significant and be part of a group
126. Rodel is very aloof and cold in his relationship with classmates. D. to get everything out in the open
Which basic goals must have not been attained by Rodel during
his developmental years, according to Erickson’s theory of 131. Teacher G’s lessons objective has something to do with the skill
psychological development? of synthesizing? Which behavioral term is most appropriate?
A. Autonomy C. Initiative A. Test C. Appraisal
B. Trust D. Generativity B. Assess D. Theorize

127. How can you exhibit expert power on the first day of school? 132. Which is a sound classroom management practice?
A. By making them feel you know what you are talking about. A. Avoid establishing routines; routines make your student robots.
B. By telling them the importance of good grades. B. Establish routines for all daily needs and tasks.
C. By reminding then your students your authority over them C. Apply rules and policies on a case to case basis.
again and again. D. Apply reactive approach to discipline.
D. By giving your students a sense of belonging and acceptance.
133. A child who gets punished for stealing candy may not steal again
128. Teacher P wants to develop the skill of synthesizing in her pupil. immediately. But this does not mean that the child may not steal
Which one will she do? again. Based on Thorndike’s theory on punishment and learning, this
A. Ask her students to formulate a generalization from the data shows that ______________.
shown in graphs. A. punishment strengthen e a response
B. Ask her students to answer questions beginning with “What if B. punishment remove a response
…” C. punishment does not remove a response
C. Tell her pupils to state data presented in graphs. D. punishment weakens a response
D. Directs her students to ask questions on the parts of the lesson
not understood. 134. Which assumption underlines the teacher’s use of performance
objectives?
52
A. Not every form of learning id observable.
B. Performance objectives assure the learner of learning. 140. With which goals of educational institutions as provided for by
the Constitution is the development of work skills aligned?
C. Learning is defined as a change in the learner’s observable A. To develop moral character
performance. B. To teach the duties of citizenship
D. The success of the learner is based on teacher’s performance. C. To inculcate love of country
D. To develop vocational efficiency
135. As a teacher, what do you do when you engage yourself in major
task analysis? 141. In instructional planning it is necessary that parts of the plan from
the first to the last have ___________.
A. Test if learning reached higher level thinking skills A. clarity C. coherence
B. Breakdown a complex task into sub-skills B. symmetry D. conciseness
C. Determine the level of thinking involved
D. Revise lesson objectives 142. All of the following describe the development of children aged
eleven to thirteen EXCEPT
136. The following are used in writing performance objectives A. they shift from impulsivity to adaptive ability
EXCEPT B. sex differences in IQ becomes more evident
A. delineate C. integrate C. they exhibit increased objectivity in thinking
B. diagram D. comprehend D. they show abstract thinking and judgement

137. Teacher B clears his throat to communicate disapproval of a 143. If a teacher plans a constructivist lesson, what will he most likely
student’s behavior. Which specific influence technique is this? do? Plan how he can _______.
A. Signal interference C. Interest boosting A. do evaluate his student’s work
B. Direct appeal D. Proximity control B. do reciprocal teaching
C. do lecture to his students
138. An effective classroom manager uses low-profile classroom D. engage his students in convergent thinking
control. What is a low-profile classroom technique?
A. Note to parents 144. In mastery learning, the definition of an acceptable standard of
B. After-school detention performance is called a
C. Withdrawal of privileges A. SMART C. behavior
D. Raising the pitch of the voice B. criterion measure D. condition

139. The primary objective of my lesson is: “To add similar fractions 145. “A stitch on time saves nine”, so goes the adage. Applied to
correctly.” Before I can do this I must first aim at this specific classroom management, this means that we _______.
objective: “to distinguish a numerator from a nominator. “What A. may not occupy ourselves with disruptions which are worth
kind of objective is the latter? ignoring because they are minor
A. Major C. Enabling B. must be reactive in our approach to discipline
B. Terminal D. Primary C. have to resolve minor disruptions before they are out of control
53
D. may apply 9 rules out of 10 consistently
152. Based on Freud’s psychoanalytic theory which component (s) of
146. Ruben is very attached to his mother and Ruth to her father. In personality is (are) concerned with a sense of right and wrong?
what developmental stage are they according to Freudian
psychological theory? A. Super ego C. Id
A. Oedipal stage C. Anal Stage B. Super-ego and Ego D. Ego
B. Latent stage D. Pre-genital stage
153. Who among the following puts more emphasis on core
147. What was the prominent educational issues of the mid 1980’s? requirements, longer school day, longer academic year and more
A. Bilingual Education C. Accountability challenging textbooks?
B. Value Education D. Mainstreaming A. Perennialist C. Progressivist
148. Which behavior is exhibited by a student who is strong in B. Essentialist D. Existentialist
interpersonal intelligence?
A. Works on his/her own. 154. A student passes a research report poorly written but ornately
B. Keeps interest to himself/herself presented in a folder to make up for the poor quality of the book
C. Seeks out a classmate for help when problem occurs. report content. Which Filipino trait does this practice prove?
D. Spends time meditating. A. art of academics
B. substance over “porma”
149. Which is behavioral term describes a lesson outcome in the C. art over science
highest level of Bloom’s cognitive domain? D. “porma” over substance
A. Create C.
155. Which one may support equitable access but may sacrifice
C. Analyze quality?
B. Evaluate D. Design A. Open admission C. Deregulated tuition fee hike
B. School accreditation D. Selective retention
150. The main purpose of compulsory study of the Constitution is to
___. 156. Based on Piaget’s theory, what should a teacher provide for
A. develop students into responsible, thinking citizens children in the sensimotor stage?
B. acquaint student with the historical development of the Phil A. Games and other physical activities to develop motor skill.
Constitution B. Learning activities that involve problems of classification and
C. make constitutional experts of the students ordering.
D. prepare students for law-making C. Activities for hypothesis formulation.
D. Stimulating environment with ample objects to play with.
151. A goal-oriented instruction culminates in _______.
A. planning activities 157. A teacher’s summary of a lesson serves the following functions,
B. evaluation EXCEPT
C. identification of topics A. it links the parts of the lesson.
D. formulation of objectives B. it brings together the information that has been discussed.
54
C. it makes provisions for full participation of students. A. coefficient of correlation C. discrimination index
D. it clinches the basic ideas or concepts of the lesson. B. central tendency D. level of difficulty

158. As a teacher, you are rationalist, Which among these will be your 165. Teacher Q does not want Teacher B to be promoted and so writes
guiding principle? an anonymous letter against Teacher B accusing her of fabricated
A. I must teach the child that we can never have real knowledge lies. Teacher Q mails this anonymous letter to School Division
of anything. Superintendent. What should Teacher Q do if she has to act
B. I must teach the child to develop his mental powers to the full. professionally?
C. I must teach the child so he is assured of heaven A. Submit a signed justifiable criticism against Teacher B, if there
D. I must teach the child every knowledge, skills and value that is any.
he needs for a better future. B. Go straight to the School Division Superintendent and gives
criticism verbally.
159. Bruner’s theory on intellectual development moves from enactive C. Hire a group to distribute poison letters against Teacher B for
to iconic and symbolic stages. In which stage(s) are diagrams the information dissemination.
helpful to accompany verbal information? D. Instigate student activists to read poison letter over the
A. Enactive and iconic C. Symbolic and enactive microphone.
B. Symbolic D. Iconic
160. Student’s scores on a test were: 72,72,73,74,76,78,81,83,85. 166. Each teacher is said to be a trustee of the cultural and educational
The score 76 is the ___. heritage of the nation and is under obligation to transmit to
A. mode C. average learners such heritage. Which practice makes him fulfill such
B. mean D. median obligation?
A. Use the latest instruction technology
161. Standard deviation is to variability as mode to _____. B. Observing continuing professional education.
A. level of difficulty C. correction C. Use interactive teaching strategies
B. discrimination D. central tendency D. Study the life of Filipino heroes

162. A teacher who equates authority with power does NOT usually 167. Which type of reports to “on-the spot” description of some
__________. incident, episode or occurrence that is being observed and
A. Shame C. develop self-respect in every pupil recorded as being of possible significance?
B. retaliate D. intimidate A. Autobiographical report
B. Value and interest report
163. Which is a true foundation of the social order? C. Biographical report
A. Obedient citizenry D. Anecdotal report
B. The reciprocation of rights and duties
C. Strong political leadership 168. Teacher A is directed to pass an undeserving student with a death
D. Equitable distribution of wealth threat. Which advice will a hedonist give?
A. Pass the student. Why suffer the threat?
164. Standard deviation is to variability as mean is to _______.
55
B. Don’t pass him. You surely will not like someone to give you
a death threat in order to pass. 173. The attention to the development of a deep respect and affection
C. Pass the student. That will be use to the student, his parents for our rich cultural past is an influence of ______
and you. A. Confucius C. Teilhard de Chardin
D. D. Don’t pass him. Live by principle of justice. You will get B. Heqel D. Dewey
reward, if not in this life, in the next.
174. A teacher / student is held responsible for his actions because s/he
169. If you agree with Rizal on how you can contribute to our nation’s _______.
redemption, which should you work for? A. has instincts C. has mature
A. Opening our doors to foreign influence B. has a choice D. is reason
B. Upgrading the quality of the Filipino through education
C. Stabilizing the political situation 175. The typical autocratic teacher consistently does the following
D. Gaining economic recovery EXCEPT
A. intimidating students C. shaming students
170. “All men are pretty much alike. It is only by custom that they are B. ridiculing students D. encouraging students
set apart, “ said one Oriental philosopher. Where can this thought
be most inspiring? 176. Teacher H and teacher I are rivals for promotion. To again the
A. In a multi-cultural group of learners. favor of the promotion staff, teacher I offers her beach resort for
B. In multi-cultural and heterogeneous groups of learners and free for members of he promotional staff before ranking. As one
indigenous peoples’ group. of the contenders for promotions, is this becoming of her to do
C. In a class composed of indigenous people. A. Yes. This will be professional growth for the promotional staff.
D. In heterogeneous class of learners. B. No. This may exert undue influence on the members of the
promotional staff and so may fail to promote on the basis of
171. In what way can teachers uphold the highest possible standard of merit.
quality education? C. Yes. The rare invitation will certainly be welcomed by an
A. By continually improving themselves personally and overworked promotional staff.
professionally D. Yes. There’s nothing wrong with sharing one’s blessings.
B. By wearing expensive clothes to change people’s poor
perception of teachers 177. Rights and duties are correlative. This means that.
C. By working out undeserved promotions A. rights and duties regulate the relationship of men in society.
D. By putting down other professions to lift the status of teaching B. rights and duties arise from natural law.
C. each right carries with it one or several corresponding duties.
172. How would you select the most fit in government position? D. rights and duties ultimately come from God.
Applying Confucius teaching, which would be the answer?
A. By course accreditation of an accrediting body 178. In the Preamble of Code of Ethics of Professional Teachers,
B. By merit system and course accreditation which is NOT said of teachers?
C. By merit system A. LET passer
D. By government examinations B. Duly licensed professionals
56
C. Possess dignity and reputation A. naturalistic morality
D. With high moral values as well as technical and professional B. classical Christian morality
competence C. situational morality
D. dialectical morality
179. What should you do if a parent who is concerned about a grade
his child received compared to another student’s grade, demands 184. What is the mean of this score distribution 4,5,6,7,8,9,10?
to see both students’ grades? A. 7 C. 8.5
A. Refuse to show either record. B. B. 6 D. 7.5
B. Show only his child’s record.
C. Refuse to show any record without expressing permission from 185. Are percentile ranks the same as percentage correct?
principal. A. It cannot be determined unless scores are given.
D. Show both records to him. B. It cannot be determined unless the number of examinees is
given
180. Teacher often complain of numerous non-teaching assignments C. No
that adversely affect their teaching. Does this mean that teachers D. Yes
must be preoccupied only with teaching?
A. Yes, if they are given other assignments, justice demands that 186. Teacher F is a newly converted to a religion. Deeply convinced
they be properly compensated. of his new found religion, he starts Monday classes by attacking
B. Yes, because other community leaders, not leaders, not one religion and convinces his pupil to attend their religion
teachers, are asked to lead in community activities. services on Sundays. Is this in accordance with the Code of
C. Yes, because every teacher is expected to provide leadership Ethics of Professional Teachers?
and initiative in activities for betterment of communities. A. Yes. What he does strengthens value education.
D. No, because teaching is enough full time job. B. No. A teacher should not use his position to proselyte others.
C. Yes. In the name of academic freedom, a teacher can decide
181. Which illustrates a development approach in guidance and what to teach.
counseling? D. Yes. What he does is a value education.
A. Spotting on students in need of guidance
B. Teaching students how to interact in a positive manner 187. In a study conducted, the pupils were asked which nationality
C. Acting as a mentor they preferred. If given a choice. Majority of the pupils wanted
D. Making the decision for the confused student to Americans. In this case, in which obligation, relative to the
state, do schools seem to be failing? In their obligation to
182. Whose influence is the education program that puts emphasis on A. respect for all duly constituted authorities
self-development through the classics, music, and ritual? B. promote national pride
A. Buddha C. Confucius C. promote obedience to the laws of the state
B. Mohammed D. Lao tsu D. install allegiance to the Constitution

183. Helping in the development of graduates who are “maka-Diyos”


is an influence of ___.
57
188. A guest in one graduation rites told his audience: “Reminder, you 193. Teacher A knows of the illegal activities of a neighbor but keeps
are what you choose to be”. The guest speaker is more of a/an quiet in order not to be involved in any investigation. Which
_____. foundational principle of morality does Teacher A fail to apply?
A. realistic C. pragmatist A. Always do what is right
B. Idealistic D. existentialist B. The principle of double effect
C. The end does not justify the means
189. From whom do we owe the theory of deductive interference as D. Between two evils, do the lesser evil
illustrated in syllogism?
A. Plato C. Socrates 194. Teacher Q does not want Teacher B to be promoted and so writes
B. Aristotle D. Pythagoras an anonymous letter against Teacher B accusing her of fabricated
lies. Teacher Q mails this anonymous letter to School Division
190. In what way can teachers uphold the highest possible standard of Superintendent. What should Teacher Q do if she has to act
quality education? professionally?
A. By continually improving themselves personally and A. Hire a group to distribute poison letters against Teacher B for
professionally the information dissemination.
B. By putting down other professions to lift the status of teaching B. Submit a signed justifiable criticism against Teacher B, if there
C. By wearing expensive clothes to change people’s poor is any.
perception of teachers C. Go straight to the School Division Superintendent and gives
D. By working out undeserved promotions criticism verbally.
D. Instigate student activists to read poison letter over the
191. Two students are given the WISC III. One has a full scale IQ of microphone
91, while the other has an IQ of 109. Which conclusion can be
drawn? 195. A teacher’s summary of a lesson serves the following functions,
A. Both students are functioning in the average range of EXCEPT
intellectual ability A. it makes provisions for full participation of students.
B. Another IQ test should be given to truly assess their B. it brings together the information that has been discussed.
intellectual potential. C. it links the parts of the lesson.
C. The first student is probably below average, while the second D. it. clinches the basic ideas or concepts of the lesson
has above average potential.
D. The second student has significantly higher intellectual ability. 196. Soc exhibits fear response to freely roaming dogs but does not
show fear when a dog is on a leash or confined to a pen. Which
192. Which describes norm-referenced grading? conditioning process is illustrated?
A. What constitutes a perfect score A. Extinction C. Generation
B. The student’s past performance B. Discrimination D. Acquisition
C. An absolute standard
D. The performance of the group 197. Ruben is very attached to his mother and Ruth to her father. In
what developmental stage are they according to Freudian
psychological theory?
58
A. Latent stage C. Anal Stage 11. C 60. D 110. C 160. D
B. Pre-genital stage D. Oedipal stage 12. D 61. B 111. D 161. D
13. C 62. C 112. C 162. C
198. The concepts of Trust vs. mistrust, autonomy vs. shame & self- 14. C 63. D 113. C 163. D
doubt, and initiative vs. guilt are most closely related with works 15. B 64. A 114. A 164. B
of _____. 16. C 65. D 115. D 165. A
A. Jung C. Erickson 17. B 66. A 116. A 166. D
B. Freud D. Piaget 18. C 67. A 117. A 167. D
19. B 68. C 118. A 168. D
199. A goal-oriented instruction culminates in _______. 20. C 69. D 119. B 169. B
A. evaluation 21. A 70. A 120. A 170 B
B. formulation of objectives 22. B 71. C 121. C 171. A
C. identification of topics 23. A 72. C 122. C 172. D
D. planning activities 24. C 73. A 123. B 173. A
25. no answer 74. A 124. A 174. B
200. Which of the following is considered a peripheral device? 26. C 75. D 125. C 175. A
A. Keyboard C. Monitor 27. A 76. B 126. B 176. B
B. CPU D. Printer 28. D 77. C 127. D 177. C
29. C 78. B 128. A 178. A
30. C 79. B 129. B 179. D
ANSWER KEY 31. A 80. C 130. C 180. C
PROFESSIONAL EDUCATION 32. C 81. C 131. D 181. A
(Secondary) 33. C 82. C 132. C 182. A
34. A 83. C 133. D 183. B
35. D 84. D 134. C 184. A
1. A 50. C 100. D 150. B 36. C 85. A 135. B 185. C
197. D 37. B 86. C 136. D 186. B
2. B 51. B 101. no answer 151. D 38. A 87. C 137. B 187. B
198. C 3. D 52. D 39. B 88. A 138. C 188. D
102. B 152. A 199. A 40 C 89. A 139. C 189. B
4. A 53. no answer 103. B 153. no 41. B 90. B 140. D 190. A
answer 200. D 42. D 91. B 141. C 191. B
5. A 54. A 104. B 154. D 43. A 92. B 142. D 192. D
6. B 55. B 105. A 155. A 44. B 93. B 143. D 193. C
7. D 56. C 106. D 156. A 45 C 94. C 144. B 194. B
8. C 57. A 107. D 157 C 46. C 95. B 145. C 195. D
9. C 58. D 108. C 158. B 47. A 96. D 146. A 196. B
10. C 59. B 109. B 159. no answer 48. A 97. D 147. B
59
49. A 98. B 148. C
99. A 149. B

60
61

Das könnte Ihnen auch gefallen